List the potential complications associated with the management of a patient after intentional corrosive ingestion.
Potential complications of intentional corrosive ingestion include:
• Acute:Oral, oesophageal, gastric bums of varying thickness Laryngeal oedema and airway obstruction Oesophageal, gastric perforation
Shock Haemorrhage Mediastinitis Psychiatric problems
• Chronic/late:
Laryngopharyngo fibrosis with airway incompetence and chronic aspiration
Oesophageal fibrosis, stricture and stenosis
Psychosocial problems
Carcinoma
This question would benefit from a systematic response.
Ramasamy, Kovil, and Vivek V. Gumaste. "Corrosive ingestion in adults." Journal of clinical gastroenterology 37.2 (2003): 119-124.
Compare and contrast the pharmacodynamics of dopamine and dobutamine.
Pharmacodynamics imply what the drug does to the body. Consideration should be given to mechanism of action, effects on various organs, relationship of dose to effect. indications for use, and type of adverse effects. These drugs are essential parts of the intensivist's armamentarium, and a good level of understanding should have been displayed.
Dopamine:
• Immediate precursor of noradrenaline, and also serves as a neurotransmitter in central and peripheral nervous systems,
• Doses of Q.5 to 2 meg/kg/min (via stimulation of DA-1 and DA-2 receptors) increase renal blood flow, urine flow and sodium excretion (inhibit sodium resorption in proximal tubules)
• Haemodynamic effects are due to noradrenaline release (up to 500/o) and direct stimulation of alpha, beta and noradrenergic receptors. Can lose effect with time (due to depletion of noradrenaline stores in periphery and heart).
• Doses of 2-5 meg/kg/min increase cardiac contractility and cardiac output with minimal change in heart rate/BP/SVR. Increasing dose up to 10 meg/kg/min increase CO/HR. and BP.
• Doses above 10 meg/kg/min result in increasing alpha adrenergic mediated vasoconstriction.
• Can increase intrapulmonary shunt (increase CO), but pulmonary vasoconstriction can occur.
• Dopamine also stimulates receptors in the zona glomerulosa of the adrenal cortex to decrease aldosterone secretion.
• A selective increase in renal and splanchnic blood flow occurs, and low doses have been thought to prevent the vasoconstrictive effects of other agents. The clinical significance of these effects are controversial (?harm to GIT via shunting from mucosa).
• Dopamine inhibits TSH and prolactin release as well as other potential negative effects on anterior pituitary function.
• Other side effects include nausea/emesis, tachyarrhythm.ias (particularly AF), anginal pain, profound vasoconstriction (including if local extravasation (treat with phentolamine]), and impairment of hypoxic ventilatory drive.
• Used to increase cardiac output and as a mild vasopressor (cardiogenic or septic shock), and as a diuretic (no evidence to support renal protective role).
Dobutamine:
• Racemic mixture of + and - isomers. + isomer stimulates both beta-adrenergic receptors. - isomer is potent selective alpha-1-ad.renergic agonist No indirect stimulation of receptors. Metabolite (3-0-methlydobutamine) is potent inhibitor of alpha receptors. Net effect is balance between various receptor effects.
• At commonly used dose ranges (2-15 meg/kg/min) increases contractility, with little effect on HR at doses < I0 meg/kg/min. Usually little effect on SVR and PVR as balance between alpha- 1 and beta-2 effects (CVP and PAWP usually decrease). Some tolerance with time but less than with dopamine.
• Enhances urine output by increasing cardiac output. No other significant metabolic or endocrine effects.
• Side effects include dysrhythmias (less than dopamine), tachycardia, headaches, anxiety, tremors, changes in BP. •
• Used to increase cardiac output without need to effect peripheral resistance (cardiogenic or septic shock) or desire to have metabolic/endocrine effects of dopamine. Also used to assess for myocardial ischaemia (stress test).
Though it may be tempting to unload a massive amount of pharmacological knowledge onto such a question, the candidate is reminded that these days we only have 10 minutes per question.
Features |
Dobutamine |
Dopamine |
Class |
Synthetic catecholamine |
Endogenous catecholamine |
Pharmacokinetics |
IV infusion 5-15mcg/kg/min Rapidly metabolised by COMT; |
Half-life 2-3minutes |
Receptor activity |
Predominantly a beta-1 receptor agonist; other isomer is a partial alpha-1 agonist |
Predominantly beta-1 receptor agonist at low doses, with more alpha-effects as dose escalates |
Mechanism |
increases heart rate and contractility by increasing the intracellular levels of cAMP, thus increasing the availablility of intracellular calcium. |
increases heart rate and contractility by increasing the intracellular levels of cAMP, thus increasing the availablility of intracellular calcium. |
Adverse effects |
Increased inotoropy |
Arrhythmogenic at the high doses required for treatment of severe sepsis Increased cardiac oxygen demand due to increased contractility and heart rate may cause ischaemic phenomena No evidence for any renal protective effects |
List the relevant pharmacology of the following drugs when used in ICU to aid the dressing of severe burns: (a) tramadol, (b) celecoxib and (c) ketamine.
Tramadol is a synthetic non-narcotic analgesic with opioid like effects. It acts centrally to bind with mu receptors and also blocks noradrenaline and serotonin uptake. It is rapidly absorbed orally with high bioavailability. It is cleared by hepatic metabolism and may produce dizziness, somnolence, nausea, constipation, sweating and pruritus similar to opioids, but causes significantly less respiratory depression than morphine.
After an IMI dose, peak effect is achieved in 45 minutes and lasts 4-5 hours. Convulsions and rare anaphylactoid reactions have been described with its use. Overdosage may produce respiratory failure and seizures. Its role in this setting is unclear as yet because of low potency but it may be useful as an adjunct.
Celecoxib is a COX-2 inhibitor and as such has anti-inflammatory, analgesic and anti-pyretic properties. In the absence of COX-1 inhibition, it should have no/little effect on gastrointestinal mucosa or platelet function. Disruption of renal blood flow autoregulation in hypovolaemia and shock is still possible.
NSAIDS have been used in burns to reduce the inflammatory response, but have an uncertain role in dressings due to slow onset (1 hour), low potency, oral preparation and untoward renal effect. Duration of action is 6 – 15 hours. They should not be used in patients with sulfonamide allergy or aspirin/NSAID associated asthma.
Ketamine is a general anaesthetic agent related to the hallucinogen phencyclidine which can be given IV or IM. Despite the tendency to emergence delirium it is a useful agent in this setting because of intense analgesia with maintenance of reflexes and minimal respiratory depression. Duration of action is 2 – 4 hours and it undergoes extensive hepatic metabolism. Dreams and hallucination can be reduced by the concomitant administration of a benzodiazapine.
Features |
Tramadol |
Celecoxib |
Ketamine |
Class/mechanism |
Partial opioid receptor agonist; |
NSAID (selctive for COX-2) |
Anaesthetic; |
Advantages |
Synergistic with other analgesics Less respiratory depression than with other opiates |
Low toxicity |
Dissociative sedation |
Disadvantages |
Weak opioid effect |
May cause renal impairment |
Confusion, delirium |
Some mixture of ketamine, tramadol and dexmedetomidine may be the best option.
Zor, Fatih, et al. "Pain relief during dressing changes of major adult burns: ideal analgesic combination with ketamine." Burns 36.4 (2010): 501-505.
Norman, Aidan T., and Keith C. Judkins. "Pain in the patient with burns." Continuing Education in Anaesthesia, Critical Care & Pain 4.2 (2004): 57-61.
Power, Camillus Kevin. "Burns Injury Pain Management-the evidence or not!."Official publication of the National Academy of Burns-India (2009).
What drug withdrawal states are relevant to ICU practice? Outline the principles of their management.
Drug withdrawal states in ICU patients may be more common than is generally appreciated. They include –
• Alcohol
• Tobacco (nicotine)
• Narcotic (heroin, morphine)
• Benzodiazepines
• Caffeine
• Other street drugs (cocaine etc)
Principles of their management include –
• prevention (avoid prolonged high dose narcotics, benzodiazepines
• detection/diagnosis (be alert for signs eg agitation, tachycardia, fever)
• sedation (may be necessary to control systemic effects)
• replacement/substitution (eg nicotine patch)
• support (airway and respiration, fluid replacement)
• simple measures such as but firm communication, reality orientation, visible clock and presence of a relative contribute to reassurance of the patient.
The following withdrawal syndromes seem relevant:
Receptor | Drugs | Clinical withdrawal syndrome | Management options |
GABAA | Alcohol Barbiturates Benzodiazepines Organic solvents |
CNS excitation (agitation, tremor, hallucinations, seizures) Autonomic stimulation (tachycardia, hypertension, hyperthermia, diaphoresis) |
Benzodiazepines Dexmedetomidine |
GABAB | GHB Baclofen |
Dyskinesia, seizures, hypertension, hallucinations, psychosis, and coma. | Benzodiazepines |
Opioid | Opiates | CNS excitation (agitation, tremor, hallucinations) Diarrhoea, mydriasis, nausea. Autonomic stimulation (tachycardia, hypertension, hyperthermia, diaphoresis) |
Clonidine Dexmedetomidine |
Adenosine | Caffeine | Head-ache (cerebral vasodilation), fatigue, and hypersomnia (motor inhibition) | - |
Nicotinic acetylcholine receptor |
Nicotine | Agitation, insomnia, poor concentration, poor gut motility, poor feed tolerance. | Varenicline? |
Noradrenenaline | Amphetamines | Agitation, dysphoria, somnolence | - |
Dopamine | Cocaine | Anhedonia, irritability, exhaustion | - |
Cannabis | Agitation, insomnia, poor gut motility | Mirtazapine ? |
The college presents an excellent summary of the generic principles of managing drug withdrawal:
In greater detail:
In this context, "prevention" is not some sort of grassroots social work movement to gets the kids off their street drugs, but rather the push towards intelligent use of opiates and benzodiazepines in the ICU. Rationalising the infusions should prevent the development of iatrogenic withdrawal syndromes. Fortunately, the ICU environment typically does not favour true psychological addiction, as the pleasurable context of drug use is not present.
In this context, detection describes vigilant monitoring for drug withdrawal:
The supportive management of drug withdrawal aims to reduce the harm from the physiological and psychological consequences of withdrawal:
The aim is to replace the drug of addiction with a less harmful substance which offers submaximal receptor stimulation, so that the symptoms of withdrawl are ameliorated and the harm of pursuing the addiction is reduced. Examples of this include methadone and varenicline.
Outline the role of decontamination of the digestive tract in the management of patients who present with a drug overdose.
Balance between potential severity of poisoning, time from ingestion and risk to the patient of interventions considered. Most overdoses do not develop significant toxicity but reasonable to use technique with low morbidity and reasonable efficacy in all except clearly non-toxic ingestions (eg. single dose activated charcoal [1g/kg]). Induced emesis with ipecac induces risks without evidence of decreased absorption. Gastric lavage is associated with reasonable decrease in absorption if performed early (e.g. < 1 hour), though it is associated with increased risks (including visceralinjury and aspiration); it may have additional benefit if combined with activated charcoal. Repeat doses of charcoal are usually not of additional benefit except perhaps where a large amount of toxic substance adsorbed by charcoal was ingested (especially slow release preparations). Whole bowel irrigation (using polyethylene glycol e.g. golytely) may have specific benefit with slow release preparations or agents that are poorly absorbed by activated charcoal. Rarely endoscopy or surgical removal is indicated.
This question closely resembles section (b) from Question 1 of the second paper of 2004. However, here it is presented on its own, as a 10-mark question, and so some extra thought should be spent on it.
In brief, decontamination can be critically evaluated in the following manner:
Rationale for decontamination
Techniques of decontamination and their indications
Situations which merit the use of gut decontamination
Criticsm of gut decontamination techniques
The website of the American Academ of Clinical Toxicology has several position statements which might be useful to the fellowship candidate:
Single-Dose Activated Charcoal
Gaudreault, Pierre. "Activated charcoal revisited." Clinical Pediatric Emergency Medicine 6.2 (2005): 76-80.
Andersen, A. Harrestrup. "Experimental Studies on the Pharmacology of Activated Charcoal. III. Adsorption from Gastro‐Intestinal Contents." Acta Pharmacologica et Toxicologica 4.3‐4 (1948): 275-284.
Krenzelok, Edward P. "New developments in the therapy of intoxications." Toxicology letters 127.1 (2002): 299-305.
Eddleston, Michael, et al. "Multiple-dose activated charcoal in acute self-poisoning: a randomised controlled trial." The Lancet 371.9612 (2008): 579-587.
Isbister, Geoffrey K., and Venkata V. Pavan Kumar. "Indications for single-dose activated charcoal administration in acute overdose." Current opinion in critical care 17.4 (2011): 351-357.
Chyka, P. A., and D. Seger. "Position statement: single-dose activated charcoal. American Academy of Clinical Toxicology; European Association of Poisons Centres and Clinical Toxicologists." Journal of toxicology. Clinical toxicology 35.7 (1996): 721-741.
Daly, F. F. S., M. Little, and L. Murray. "A risk assessment based approach to the management of acute poisoning." Emergency medicine journal 23.5 (2006): 396-399.
Olmedo, Ruben, et al. "Is surgical decontamination definitive treatment of “body-packers”?." The American journal of emergency medicine 19.7 (2001): 593-596.
Compare and contrast the pharmacology of lignocaine, magnesium and amiodarone when used in the treatment of ventricular tachycardia.
• Lignocaine: Class I (membrane stabilising) antiarrhythmic agent. Sodium channel blockades results in decreased action potential duration and shortened refractory period. Rapidly distributed to all body tissues. Approximately 65% protein bound; elimination half-life 1.6 hours (80% metabolised in liver). Adverse effects: lightheaded, hypotension, cardiovascular collapse, heart block, confusion and convulsions. Dosage 1 to 1.5mg/kg with subsequent boluses (up to 3 mg/kg total), followed by infusion (1-4 mg/min, at decreasing dose, up to
24 hours).
• Magnesium (as sulphate or chloride): second most abundant intracellular cation. Depresses neuronal activation. Widely distributed, duration of action about 30 minutes. Filtered by kidneys, but most is reabsorbed. Adverse effects include: nausea, flushing, CNS depression, coma, and heart block. Dose 5 mmol bolus (which may be repeated), followed by infusion
of 20 mmol over 4 hours.
• Amiodarone: Class III antiarrhythmic. Prolongs action potential duration, and prolongs refractory period of atrial, nodal and ventricular tissues. Highly protein bound with very high apparent volume of distribution (6 L/kg); accumulates in adipose tissue and highly perfused organs. Half-life (with chronic dosing) is 14 to 59 days, mainly excreted via the liver and bile. Adverse effects: hypotension/circulatory collapse, bradycardia, sinus arrest, nausea and flushing. Torsades de pointes can be induced. Hyper- or hypo-thyroidism can be
induced. Multiple other potential organ dysfunctions with more chronic use (including some potentially fatal). Dosage 5 mg/kg which can be repeated, and followed by an infusion (15 mg/kg/hr).
This question is identical to Question 12 from the first paper of 2006.
Compare and contrast the pharmacology of propofol, midazolam and thiopentone when used by infusion for the treatment of raised intra-cranial pressure.
Pharmacology includes pharmaceutics (including preparation), pharmacokinetics (including distribution, elimination and biotransformation) and pharmacodynamics (including dose, mechanism of action, effect of various disease states, adverse effects and interactions). Effects of all drugs are augmented by other CNS depressant drugs.
Propofol is formulated as a white isotonic aqueous emulsion (containing soya oil and egg lecithin) at a concentration of 10 mg/mL. It supports the growth of bacteria if accidentally introduced so syringes/bottles should be changed every 12 hours. It is widely and rapidly distributed (98% protein bound), with an initial half life of redistribution of 2 to 8 minutes, but a terminal elimination of 3 to 20 hours (which may influence waking time after prolonged infusion [many days]: context sensitive half-time). Inactive metabolites are renally excreted. Usual ICU sedation dose is infusion of 1 to 3 mg/kg/hr (higher infusion rates in Intensive Care have been associated with rhabdomyosysis). Mechanism of action is not clear. Lipid formulation of approx. 1 kcal/mL should be taken into account, and high triglyceride levels may be seen in susceptible patients. Hypotension and depression of cardiac output may occur (more so when bolus doses are used). Compatible with
5% dextrose but not with many other solutions or drugs. Time to wake after cessation of infusion is short (minutes to hours) depending on duration of infusion (context sensitive half time). Rapid awakening may increase likelihood of convulsions in those susceptible. Relatively expensive.
Midazolam is formulated as a colourless isotonic but acidic solution (pH 3.3) as 1 or 5 mg/mL. Onset usually seen within minutes (97% protein bound), and usual elimination half-life quoted at 1 to 3 hours, but often seen 6 times longer in critically ill especially elderly or renal failure Metabolised by P450-3A to active metabolite 1-OH-methyl midazolam, and then renally excreted. Usual ICU dosage 0.03 to 0.2 mg/kg/hr. Mechanism of action is via activation of benzodiazepine receptor, which augments the inhibitory effect of the GABA receptor. Cardiorespiratory depression is expected. Rapid cessation may lead to withdrawal. Compatible with many solutions and drugs (except hartmanns), infusions should be discarded after 24 hours. Time to wake after cessation of infusion is intermediate (hours to days) depending on duration of infusion and presence of renal or hepatic dysfunction.
Thiopentone is prepared from a powder and dissolved in water giving an alkaline solution with a final concentration of 25 mg/mL. Onset of action within minutes (80% protein bound, with very large volume of distribution), usual elimination half life quoted at 3 to 8 hours (but presumably longer after prolonged infusion). Metabolised predominantly in the liver to (?) inactive metabolites which are renally excreted. Usual ICU dosage is 25 to 100 mg/hour (0.5 to 1.5 mg/kg/hr), with boluses of 25 to 100 mg as required. Very effective CNS depressant (including resultant isoelectric EEG and fixed dilated pupils). Incompatible with many solutions (especially if acidic), and infusions should be discarded after 24 hours. Time to awakening after cessation of infusion is delayed (up to many days), depending on duration of infusion and the presence of hepatic dysfunction. Barbiturates may precipitate acute porphyria in susceptible patients by enhancing porphyrin synthesis.
Features |
Propofol |
Midazolam |
Thiopentone |
Class |
General anaesthetic phenol |
Benzodiazepine |
barbiturate |
Pharmacokinetics |
Half-life 2-3minutes Extensive distribution into fat |
Half-life 90 minutes; |
Half life ~ 9 hours; rapid redistribution decreases the duration of effect. Extensive distribution into fat |
Receptor activity |
Thought to be a GABA agonist; |
Allosteric modulator of GABA neurotransmission; increases the rate of opening of GABA channels |
Allosteric modulator of GABA neurotransmission; increases the duration of opening of GABA channels |
Advantages in head injury |
Short half life Decreases cerebral metabolic demand, thus decreasing ICP Decreases seziure activity |
Decreases cerebral metabolic demand, thus decreasing ICP Decreases seziure activity |
Decreases cerebral metabolic demand, thus decreasing ICP Decreases seziure activity (very potent antiepileptic) Well tolerated in hemodynamic instability |
Disadvantages in head injury |
Hypotension |
Slow waking; context-sensitive half-life becomes prolonged Sudden withdrawal may precipitate seziures Prolonged effect in renal or hepatic impairment |
Very slow waking; as drug redistributes from tissue compartment May precipitate acute porphyria |
Outline the diagnostic features, complications and treatment of patients with an overdose of sodium valproate (valproic acid).
Sodium valproate is becoming more widely used (seizures, bipolar disorders, migraine), and is often prescribed as a slow release preparation. Overdose results in a progressive onset of lethargy and CNS depression, with many potential associated features (including hypotension, hypothermia, vomiting, diarrhoea, agitation and tremors). Complications include cerebral oedema (with prolonged coma), encephalopathy (elevated ammonia), hepatotoxicity (rarely fulminant), and electrolyte disorders (with hypernatraemia, hypocalcaemia, increased osmolality and elevated anion gap metabolic acidosis). Treatment is generally supportive but gastrointestinal decontamination is essential (including multiple dose activated charcoal &/or whole bowel irrigation if sustained release preparations, and increasing valproic acid levels). Carnitine supplementation may attenuate hepatotoxicity and hyper-ammonaemia.
This is hard, because there are no characteristic clinical features in this overdose. There is non-specific lethargy which progresses to drowsiness and coma. Then, the LFTs come back deranged, and when you do the ammonium level it is through then roof, which makes you think.
Pathophysiology
Diagnostic features
Complications of valproate overdose
Drug levels
Treatment
One report linked below is an account of a truly massive (25g) valproate overdose, which did not require anything but supportive management, and which was not accompanied by any sort of massive organ system failure.
Isbister, Geoffrey K., et al. "Valproate overdose: a comparative cohort study of self poisonings." British Journal of clinical pharmacology 55.4 (2003): 398-404.
Lakhani, Mayur, and M. E. McMurdo. "Survival after severe self poisoning with sodium valproate." Postgraduate medical journal 62.727 (1986): 409-410.
Löscher, Wolfgang. "The discovery of valproate." Valproate. Birkhäuser Basel, 1999. 1-3.
Licari, Elisa, et al. "Life-threatening sodium valproate overdose: A comparison of two approaches to treatment*." Critical care medicine 37.12 (2009): 3161-3164.
Lheureux, Philippe ER, et al. "Science review: Carnitine in the treatment of valproic acid-induced toxicity–what is the evidence?." Critical Care 9.5 (2005): 431.
Compare and contrast the pharmacology of ceftriaxone, gentamicin and meropenem.
Ceftriaxone: vial with yellow water soluble powder for reconstitution; only administered parenterally, 33-66% excreted unchanged in urine, no active metabolites, 85-95% protein bound, elimination half life 6-9 hours (> 36 hours with severely impaired renal function), usual dosage 0.5 to 2g IV 12 or 24 hourly; 3rd generation cephalosporin antibiotic, inhibits cell wall synthesis, covers most gram negative rods (except Pseudomonas), and Gram positive cocci (except Methicillin Resistant, and group D streptococci); adverse reactions uncommon, but include overgrowth of non- susceptible organisms, and occasional haematologic, renal and hepatic adverse effects.
Gentamicin: ampoule with 80 mg/2 mL; only administered parenterally, excreted almost entirely by glomerular filtration, elimination half life 2-3 hours, no active metabolites, usual dosage 1 mg/kg tds or up to 5 mg/kg as daily dose, careful monitoring of blood levels required, especially if renal impairment (trough level not > 2 mcg/mL); aminoglycoside antibiotic, inhibits protein synthesis, covers most gram negative rods (including pseudomonas, but variability from hospital to hospital); serious adverse reactions include oto- and renal toxicity, potentiated by other oto- and nephro- toxins, prolongation of neuromuscular blockade may occur, other reactions uncommon.
Meropenem: vial with water soluble powder for reconstitution; only administered parenterally, 70% excreted unchanged in urine (requiring reduction of dosage if significant renal impairment), plasma binding 2%, elimination half life 1 hour, no active metabolites, usual dosage 500mg to 2g every 8hours; carbapenem antibiotic, inhibits cell wall synthesis, active against a broad spectrum of aerobic and anaerobic bacteria (including Gram positive cocci and Gram negative rods, but excluding MRSA, Enterococcus faecium, Sternotrophomonas and many Pseudomonas); serious adverse reactions are rare, but include overgrowth of non-susceptible organisms, and occasional haematologic, gastrointestinal and hepatic adverse effects.
This question is identical to Question 3 from the second paper of 2006.
Outline the diagnostic features, complications and treatment of patients with malignant hyperpyrexia.
Malignant hyperpyrexia is a rare genetic disorder, usually autosomal dominant inheritance, with mutations of the calcium channel (ryanodine) found in the sarcoplasmic reticulum of skeletal muscle. When triggered by drugs (esp. suxamethonium and volatile anaesthetic agents), usually within 1 hour, uncontrolled calcium efflux results in tetany, and markedly increased skeletal muscle metabolism.
Diagnostic features include susceptible patient (may be unknown), exposed to triggering agent, with signs of increased metabolic rate (early tachycardia, increased muscle tone, increased oxygen consumption, increased CO2 production [e.g. ETCO2], and later marked hyperthermia). Complications include rhabdomyolyis, shock, disseminated intravascular coagulation, and a mixed metabolic (lactic) and respiratory acidosis.
The mainstay of treatment is the removal of triggering agents and administration of the specific antidote (dantrolene 20 mg/vial, diluted to 60 mL with water, dosage e.g. 2 mg/kg every 5 minutes up to 10 mg/kg, repeated every
10 to 15 hours, and continued for three days). Other treatment is supportive initially with active cooling, and detection and treatment of the potential complications listed above. Confirmation of diagnosis (muscle biopsy) and family screening may be necessary.
The European Malignant Hyperthermia Group has published some nice guidelines in 2010, which offer an excellent overview of this topic.
General features
Clinical features
Complications
Management
Hopkins, P. M. "Malignant hyperthermia: advances in clinical management and diagnosis." British journal of Anaesthesia 85.1 (2000): 118-128.
Glahn, K. P. E., et al. "Recognizing and managing a malignant hyperthermia crisis: guidelines from the European Malignant Hyperthermia Group." British journal of anaesthesia 105.4 (2010): 417-420.
Compare and contrast the pharmacology of noradrenaline, vasopressin and phenylephrine
Noradrenaline is the catecholamine released by postganglionic adrenergic nerves. Direct agonist acting on alpha (vasoconstrictor: arterial and venous) and beta-1 (contractility, pro-arrhythmic) adrenergic receptors. Not absorbed enterally. Rapidly metabolised by COMT and MAO, resulting short (minutes) duration of effect (usually administered as intravenous infusion into central vein at rate of 0.5 to 100 mcg/min). Used clinically to increase blood pressure (usually in the setting of vasodilatory shock).
Vasopressin is a hormone/neurotranmitter with a complex series of effects. Direct action on a number of receptors (V1 (vascular: vasoconstriction), V2 (renal: anti-diuresis), V3 (pituitary), OTR (oxytocin receptor subtypes) and P2 (purinergic). Not absorbed enterally. Rapidly inactivated by trypsin and peptidases, resulting in short (minutes) duration of effect (longer on kidneys as very low concentration are required). Used clinically as treatment for diabetes insipidus (IM, IV or intranasal), and more recently by intravenous infusion (via central vein at rates of 0.01 to 0.1
U/min) to increase blood pressure (usually in the setting of vasodilatory shock) or as a large intravenous bolus providing potent vasoconstriction during cardiac arrest (40 units). Potentiates the action of other vasoconstrictor agents.
Phenylephrine is a synthetic alpha-1 adrenoreceptor agonist, similar in structure to adrenaline. Not administered enterally, biotransformation not well described but duration of action longer than naturally occurring catecholamines (still minutes). Used clinically for vasoconstrictor effects, usually administered intravenously either in small bolus doses or occasionally as an intravenous infusion (via a central vein at rates of 40 to 180 mcg/min). Refractory hypotension may respond to agents with combined alpha-1 & alpha-2 activity (e.g. noradrenaline). Can be administered topically for alpha-adrenergic effect.
Of these "compare and contrast" questions, vasopressin noradrenaline adrenaline phenylephrine dobutamine levosimendan and dopamine have all been asked about.
In fact, even this current mixture has cropped up in Question 18 of the second paper from 2005.
You are called to see a 16-year-old girl in the Emergency Department. She was brought in by ambulance after being found unconscious by her parents. She was last seen alive and well 12 hours ago. Several empty bottles of tablets were found beside her.
(c) What “antidotes” are available for patients after drug overdose?
(c) What “antidotes” are available for patients after drug overdose?
Many antidotes are available but obviously their relevance depends on the clinical scenario and the specifics of the drugs ingested. Specific antidotes for commonly used agents (eg. naloxone for opioids, flumazenil for benzodiazepines, beta-agonists for beta-blockers, Ca for calcium channel blockers, protamine for heparin, atropine for organophosphates, and physostigmine for anticholinergics). Less commonly used specific antidotes include:
digibind for digoxin, and desferrioxamine for iron. Other indirectly acting antidotes include: Fresh Frozen Plasma and Vitamin K for warfarin, N-acetyl cysteine for paracetamol, glucagon for beta- and calcium channel blockers, glucose for insulin, ethanol for methanol, sodium bicarbonate for tricyclic antidepressants and praladoxime for organophosphates.
c)
This question closely resembles Question 28.1 from the second paper of 2009, as well as Question 14.2 from the first paper of 2008 and Question 2 from the first paper of 2007. There are so many lists of antidotes available that I see no point in repeating this answer endlessly.
The website of the American Academ of Clinical Toxicology has several position statements which might be useful to the fellowship candidate:
You are called to see a 16-year-old girl in the Emergency Department. She was brought in by ambulance after being found unconscious by her parents. She was last seen alive and well 12 hours ago. Several empty bottles of tablets were found beside her.
(d) Discuss her ongoing (definitive) management.
(d) Discuss her ongoing (definitive) management.
Definitive management of this girl includes specifics related to the drugs involved (eg. antidotes listed above for paracetamol or tricyclic antidepressants; continuation or otherwise of decontamination techniques) or the presence of any intercurrent diseases (eg. rhabdomyolysis). General supportive care would include attention to pressure areas, nutrition, thromboprophylaxis, and nosocomial infections. Specific care would be directed
to parents/relatives, and psychiatric assessment is required early to facilitate appropriate psychiatric management.
d)
The specific management of this overdose victim would depend completely on the drug overdosed upon. All one can say is motherhood statements about supportive management, be it ventilation, sedation, administration of various antidotes, dialysis, vasopressor support, correction of acid-base abnormalities, and councelling of the parents.
In short:
Risk assessment
Screening investigations:
Enhanced elimination
Supportive ICU management
A) - If in doubt, keep them intubated.
B) - Keep them ventilated with a mandatory mode initially; ensure that the minute volume is enough to help them compensate for the acidosis they were experiencing. Classically, the patients with salicylate overdose end up dying suddenly if they are ventilated slowly, and the ensuing respiratory acidosis improves the lipid solubility and CNS penetration of their serum salicylate. Specific strategies may apply in certain circumstances, particularly in the case of paraquat toxicity (where oxygen has a known deleterious effect)
C) - haemodynamic support as required - this may range from ECMO to beta blockade and nitroprusside
D) - nothing specific can be said except the use of benzodiazepines is encouraged in the literature, both as a means of seizure prophylaxis and as a means of controlling a potential impending withdrawal syndrome. Practically, long-acting benzodiazepines are not desirable, as they obscure the neurological findings.
E) - Normal electrolyte concentrations protect the patient from such badness as torsade
F) - Forced diuresis may not be indicated for virtually any intoxication apart from perhaps cyclophosphamie, or in the case of rhabdomyolysis. However, maintaining a good urine output promotes renal clearance of drugs which benefit from it.
G) - There is rarely a firm contraindication to nutrition
H) - There is rarely a requirement for transfusion, but exchange transfusion is a possible solution to severe methaemoglobinaemia.
I) - antibiotics are rarely required; extremes of temperature may require cooling or heating.
The website of the American Academy of Clinical Toxicology has several position statements which might be useful to the fellowship candidate:
You are called to see a 16-year-old girl in the Emergency Department. She was brought in by ambulance after being found unconscious by her parents. She was last seen alive and well 12 hours ago. Several empty bottles of tablets were found beside her.
(a) What is your initial management?
(a) What is your initial management?
Initial management is to and assess vital signs (airway, breathing and circulation), institute appropriate monitoring (ECG, pulse oximeter) and institute whatever immediate supportive management is required. Early supportive management of the airway and breathing may require endotracheal intubation (eg. significant hypoxia, GCS < 9, not protecting airway, respiratory acidosis), and circulation will normally require intravenous fluids and/or vasopressors (ie. intravenous ± central venous access). History of presentation (including nature of tablets found and other medications she would have access to), past history of medical problems (including treatment and allergies) and time course of presentation are essential (from whoever can provide the most information). Examination allows search for toxidromes (pupils, sweating, heat rate etc), focal neurological signs (which may suggest an alternate diagnosis) and any complications of unconsciousness including aspiration, pressure areas etc.) Early investigations would include blood gases (oxygenation, ventilation, acidosis), electrolytes (especially K), blood glucose and paracetamol levels (treatable problem). Other specific investigations may be indicated (eg CK, Creatinine, phosphate if concerned about rhabdomyolysis; osmolality for osmolar gap etc.). It would be reasonable to consider a head CT if there were concerns about the neurological state. Decontamination
and antidotes are considered in subsequent parts of this question.
a)
The website of the American Academ of Clinical Toxicology has several position statements which might be useful to the fellowship candidate:
Compare and contrast the pharmacology of ketamine, morphine and dexmetatomidine when used for analgesia in the critically ill.
Ketamine is a non barbiturate general anaesthetic, produces a state of “dissociative anaesthesia” with profound analgesia. Pharmaceutics: racemic mixture , clear liquid in ampoule with 200mg in 2mL. Pharmacokinetics: initial rapid redistribution (T 1/2 10 to 15 minutes) representing anaesthetic action, followed by beta phase half life of about 2.5 hrs. 2-
50% protein bound. Volume of distribution 1.8 L/kg, 90% excreted by urine (mainly after extensive hepatic metabolism to less active metabolites, only 4% unchanged). Can be administered IV, IM or SC. Pharmacodynamics: onset of action within 30 seconds and duration of analgesia approximately 30 minutes (profound analgesia of shorter duration). Relative preservation of respiratory reflexes (except at higher dosages), can increase BP and ICP, and result in involuntary movements and emergent reactions. Anaesthetic doses 1-2 mg/kg, but analgesia can be obtained with lower doses (eg. 10-20 mg; 0.1-0.3 mg/kg) or by low dose infusion (eg. 0.1 mg/kg/hr). Value if need short periods of profound analgesia.
Morphine is an opioid analgesic which activates predominantly mu opioid receptors. Pharmaceutics: clear liquid in ampoule with 10 (or 15) mg in 1 mL. Pharmacokinetics:
initial rapid redistribution, followed by more prolonged elimination phase half life of about
2 hrs. 35% protein bound. Volume of distribution 3.3 L/kg, 90% excreted by urine (mainly after hepatic metabolism to active metabolite morphine-6-glucuronide which has a longer half life). Can be administered IV, IM or SC. Pharmacodynamics: rapid onset of action when injected intravenously and duration of analgesia dose (up to hours). Effects significantly prolonged with hepatic or renal dysfunction. Adverse effects include hypotension, sedation, and significant depression of respiratory and gastrointestinal
function, and rarely “biliary” spasm. Antagonist exists: naloxone. Dosage: 10 mg IM, or 1-2 mg boluses (eg. with PCA) and infusion of 1-5 mg/hr.
Dexmedetomidine is a relatively selective alpha-2 adrenoreceptor agonist (providing its sedative and analgesic effects). Pharmaceutics: expensive, clear liquid in ampoule with
200mcg in 2 mL. Pharmacokinetics: initial rapid redistribution (six minutes), followed by more prolonged elimination phase half life of about 2 hrs. 94% protein bound. Volume of distribution 1.5 L/kg. Near complete hepatic metabolism to inactive metabolites which are then excreted in the urine. Administration only by IV infusion (load of 1 mcg/kg, followed by 0.2 to 0.7 mcg/kg/hr). Effects may be prolonged with hepatic or renal dysfunction. Predominant adverse effects include hypotension, bradycardia (including sinus arrest) and dry mouth. Predominant use is for profound sedation for short periods (eg. 24 hours).
First of all, let me waggle a shaming finger at the misspelling of dexmedetomidine. It is, of course, possible that some underpaid typist was charged with writing up the college paper from dictated notes, if the examiners in 2004 were loath to soil their carefully manicured hands with the crudeness of keyboard work. Nevermind. At least it is correct in the answer.
Features |
Ketamine |
Morphine |
Dexmedetomidine |
Class |
NMDA receptor antagonist |
Opiate |
Central Alpha-2 agonist |
Pharmacokinetics |
Multiple possible routes of administration Half life 0.5-2hrs Hepatic metabolism into weakly active metabolites, which are cleared renally |
Multiple possible routes of administration Half-life 4-9 hours, however has a rapid initial redistribution. Hepatic metabolicm into active metabolites, which are cleared renally |
IV administration Half-life 20 minutes Hepatic metabolism into inactive metabolites |
Receptor activity |
NMDA receptor antagonist |
opiate receptor agonist |
Central alpha-2 receptor agonist |
Mechanism |
Competitive antagonism of glutamate neurotransmission at the NMDA receptor |
Activates opioid receptors, which activatenumerous intracellular signalling pathways including decreasing the intracellular cAMP levels and closing voltage-sentitive calcium channels, overall leading to decreased neuronal excitability |
Acts on presynaptic alpha-2 receptors to decrease the release of synaptic noradrenaline in the central nervous system, thereby decreasing sympathetic outflow. |
Benefits in analgesia |
Reduces opiate requirements Effective co-analgesic Effective for procedural analgesia and sedation |
Strong analgesic Decreases sympathetic responses to pain Decreases sedation requiremens Improves tolerance of uncomfortable features of ICU stay (eg. ETT, central lines) |
Reduces opiate requirements Effective co-analgesic Effective for procedural analgesia and sedation Additional benefit of sedation without respiratory depression |
Adverse effects |
Confusion |
Confusion |
Bradycardia |
You are called to see a 16-year-old girl in the Emergency Department. She was brought in by ambulance after being found unconscious by her parents. She was last seen alive and well 12 hours ago. Several empty bottles of tablets were found beside her.
(b) What is the role of decontamination of the digestive tract?
(b) What is the role of decontamination of the digestive tract?
The role of decontamination of the digestive tract is controversial. This does not refer to Selective Decontamination of the Digestive tract (SDD) which is a form of antimicrobial prophylaxis. The induction of emesis is not favoured. The routine use of gastric lavage and/or activated charcoal has lost favour in the majority of overdose situations because of the limited evidence of benefit, and the possibility of harm (eg. aspiration or trauma). There are some situations where either or both of these techniques should be considered: early presentation (eg. < 1 hour) or presence of a drug which would delay gastric emptying, and presence of toxic drug in high quantities (eg. lethal dose) especially if in a slow release form. Administration of charcoal does not absorb small highly ionised chemicals (eg. metals, electrolytes, acids and alkali). Additional techniques such as repeated activated charcoal (and/or cathartics eg. sorbitol) or whole bowel irrigation (eg. with polyethylene glycol balanced electrolyte solution) may be considered (especially with slow release preparations). Rarely is surgical removal required.
b)
Rationale for decontamination
Techniques of decontamination and their indications
Situations which merit the use of gut decontamination
Criticsm of gut decontamination techniques
The website of the American Academ of Clinical Toxicology has several position statements which might be useful to the fellowship candidate:
List potential adverse drug reactions, and outline how they may impact on your management of the critically ill patient.
This question was best answered using a systematic approach. Many candidates did not address the impact of adverse reactions on management, in particular how to prevent or minimise their occurrence. Reasonable list should include effects of drug alone and drug on drug.
Expected reactions (ie extensions of known pharmacologic effects) are many and should include pharmaceutic (eg. compatibility issues), pharmacokinetic (eg. absorption, enzyme induction), pharmacodynamic (eg. innocent bystander organs, competition). Unexpected reactions include idiosyncratic (haematological, hepatic, dermatological), and allergic (mild through to anaphylaxis and anaphylactoid). Management requires detailed drug history: drugs administered (over the counter as well as prescription drugs), alcohol intake, previous drug reactions, conditions that make adverse effects more likely (eg. respiratory depression and sleep apnoea, or severe airways disease). Examination: to look for conditions that may make reactions more likely. Careful prescribing (ie only using drugs when indicated) with attention to potential interactions (including physical incompatibilities etc), and appropriate monitoring (eg. drug levels, organ function).
Twenty-three out of forty-one candidates passed this question.
This is a question which would benefit from a systematic approach. An alternative to the college system is the one offered in Table 1 from the Lancet article by Edwards et al (2000). Apparently, that is the official classification used in "the industry".
Dose-related reactions
Non-dose-related reactions
Dose and time related reactions
Time related reactions
Withdrawal reactions
Unexpected failure of therapy
Immediate management:
Investigation
Prevention
Leape, Lucian L., et al. "Pharmacist participation on physician rounds and adverse drug events in the intensive care unit." Jama 282.3 (1999): 267-270.
Edwards, I. Ralph, and Jeffrey K. Aronson. "Adverse drug reactions: definitions, diagnosis, and management." The Lancet 356.9237 (2000): 1255-1259.
Cullen, David J., et al. "Preventable adverse drug events in hospitalized patients: a comparative study of intensive care and general care units." Critical care medicine25.8 (1997): 1289-1297.
Bates, David W., et al. "Incidence of adverse drug events and potential adverse drug events: implications for prevention." Jama 274.1 (1995): 29-34.
Compare and contrast the pharmacology of noradrenaline, vasopressin and phenylephrine when used as vasopressors in the critically ill.
Noradrenaline is the catecholamine released by postganglionic adrenergic nerves. Direct agonist acting on alpha (vasoconstrictor: arterial and venous) and beta-1 (contractility, pro-arrhythmic) adrenergic receptors. Not absorbed enterally. Rapidly metabolised by COMT and MAO, resulting short (minutes) duration of effect (usually administered as intravenous infusion into central vein at rate of 0.5 to 100 mcg/min). Used clinically to increase blood pressure (usually in the setting of vasodilatory shock).
Vasopressin is a hormone/neurotranmitter with a complex series of effects. Direct action on a number of receptors (V1 (vascular: vasoconstriction), V2 (renal: anti-diuresis), V3 (pituitary), OTR (oxytocin receptor subtypes) and P2 (purinergic). Not absorbed enterally. Rapidly inactivated by trypsin and peptidases, resulting in short (minutes) duration of effect (longer on kidneys as very low concentration are required). Used clinically as treatment for diabetes insipidus (IM, IV or
intranasal), and more recently by intravenous infusion (via central vein at rates of 0.01 to 0.1
U/min) to increase blood pressure (usually in the setting of vasodilatory shock) or as a large intravenous bolus providing potent vasoconstriction during cardiac arrest (40 units). Potentiates the action of other vasoconstrictor agents.
Phenylephrine is a synthetic alpha-1 adrenoreceptor agonist, similar in structure to adrenaline. Not administered enterally, biotransformation not well described (not metabolised by COMT) but duration of action longer than naturally occurring catecholamines (still minutes). Used clinically for vasoconstrictor effects, usually administered intravenously either in small bolus doses or occasionally as an intravenous infusion (via a central vein at rates of 40 to 180 mcg/min).
Refractory hypotension may respond to agents with combined alpha-1 & alpha-2 activity (e.g. noradrenaline).
Great masses of text in homage to noradrenaline and vasopressin are available elsewhere. Phenylephrine is less common.
Features |
Noradrenaline |
Phenylephrine |
Vasopressin |
Class |
Endogenous catecholamine |
Synthetic catecholamine |
Endocrine nonapeptide |
Pharmacokinetics |
Half-life 2-3minutes |
Half-life 5-10 minutes |
0.002 units /kg/min; or, 2-2.4 units/hr |
Receptor activity |
Predominantly alpha-1 agonist activity; Affinity for receptors decreases in acidosis |
Strongly selective for alpha-1 receptors Affinity for receptors decreases in acidosis |
Acts on V1 receptors (for vasopressor activity) and on V2 receptors (for antidiuretic activity). |
Mechanism |
Increases intracellular IP3, which in turn increases the availablility of intracellualr calcium to smooth muscle contractile proteins |
Increases intracellular IP3, which in turn increases the availablility of intracellualr calcium to smooth muscle contractile proteins |
V1 effect is by Gq-protein coupled receptors, which also increases intracellular IP3. |
Clinical effects |
Arterial and venous vasoconstriction |
Arterial and venous vasoconstriction |
Arterial and venous vasoconstriction |
For those (like me) who are unfamiliar with phenylephrine, there is an excelent monograph on its properties published by Schering-Plough (it is the Briefing Document for NDAC Meeting in December 14, 2007)
Outline the clinical features, mechanism of toxicity and therapy of cyanide poisoning.
Clinical features:
Symptoms of toxicity range from non-specific symptoms such as headache and nausea to depressed consciousness, seizures and cardiopulmonary arrest. Laboratory features include lactic acidosis and unexpectedly high venous oxygen saturation (with low a-v oxygen difference)
Mechanism of toxicity:
Cyanide blocks mitochondrial cytochrome oxidase resulting in cytotoxic hypoxia and lactic
acidosis.
Therapy:
As cyanide is highly toxic and can penetrate intact skin or be inhaled. Consequently decontamination is essential and mouth-to-mouth resuscitation should not be performed. In cases of ingestion gastric lavage may reduce absorption.
There are various antidotes based on three principles:
1. Conversion of haemoglobin to methaemoglobin (Amyl nitrite or sodium nitrite are used for this purpose). Methaemoglobin has a higher affinity for cyanide than does cytochrome oxidase and therefore promotes its dissociation from cytochrome oxidase. Since methaemoglobin does not carry oxygen, excessive methaemoglobinaemia can lead to anoxia. Methaemoglobin should be measured during treatment; a desirable level is between 20% and 30%.
2. Direct binding to EDTA or the vitamin B12 precursor hydroxocobalamin. A high dose (5 grams)
of hydroxocobalamin is required but has minimal toxicity (in contrast to other treatments).
3. Thiosulfate (administered as sodium thiosulphate) reacts with cyanide forming the relatively non- toxic thiocyanate, which is excreted in the urine. This action is slow and provides little effect in the acute phase.
This one is among my favourites.
Supportive management
Antidotes:
Hall, Alan H., and Barry H. Rumack. "Clinical toxicology of cyanide." Annals of Emergency Medicine 15.9 (1986): 1067-1074.
Beasley, D. M. G., and W. I. Glass. "Cyanide poisoning: pathophysiology and treatment recommendations." Occupational medicine 48.7 (1998): 427-431.
Mutlu, Gökhan M., et al. "An unresponsive biochemistry professor in the bathtub." CHEST Journal 122.3 (2002): 1073-1076.
Cummings, T. F. "The treatment of cyanide poisoning." Occupational Medicine 54.2 (2004): 82-85.
Wesson, D. E., et al. "Treatment of acute cyanide intoxication with hemodialysis." American journal of nephrology 5.2 (1985): 121-126.
Compare and contrast the pharmacology of lignocaine, magnesium and amiodarone when used in the treatment of ventricular tachycardia.
Lignocaine:
Class I (membrane stabilising) antiarrhythmic agent. Sodium channel blockades results in decreased action potential duration and shortened refractory period. Rapidly distributed to all body tissues. Approximately 65% protein bound; elimination half-life 1.6 hours (80% metabolised in liver). Adverse effects: lightheaded, hypotension, cardiovascular collapse, heart block, confusion and convulsions.
Dosage used in the treatment of ventricular tachycardia: 1 to 1.5mg/kg with subsequent boluses (up to 3 mg/kg total), followed by infusion (1-4 mg/min, at decreasing dose, up to 24 hours).
Magnesium (as sulphate or chloride):
Second most abundant intracellular cation. Depresses neuronal activation. Widely distributed, duration of action about 30 minutes. Filtered by kidneys, but most is reabsorbed.
Adverse effects include: nausea, flushing, CNS depression, coma, and heart block.
Dose used in the treatment of ventricular tachycardia: 5 mmol bolus (which may be repeated), followed by infusion of 20 mmol over 4 hours.
Amiodarone:
Class III antiarrhythmic. Prolongs action potential duration, and prolongs refractory period of atrial, nodal and ventricular tissues. Highly protein bound with very high apparent volume of distribution (6 L/kg); accumulates in adipose tissue and highly perfused organs. Half-life (with chronic dosing) is 14 to 59 days, mainly excreted via the liver and bile.
Adverse effects: hypotension/circulatory collapse, bradycardia, sinus arrest, nausea and flushing. Torsades de pointes can be induced. Hyper- or hypo-thyroidism can be induced. Multiple other potential organ dysfunctions with more chronic use (including some potentially fatal).
Dosage used in the treatment of ventricular tachycardia: 5 mg/kg (or 300 mg in adults) which can be repeated, and followed by an infusion (15 mg/kg/hr).
Again, the college answer outlines all the important stuff.
The table below reconfigures this into a more eye-pleasing form.
Features |
Lignocaine |
Magnesium |
Amiodarone |
Class |
Class 1a antiarrhytmic |
Divalent cation |
Class 3 antiarrhytmic (though it has effects of all 4 classes) |
Administration / dosage |
IV Then, 4 mg/kg for the first hour, then tapering infusion to 1mg/kg for 24 hrs |
IV 10-20mmol/L given over 15-60 minutes, or 5 mmol boluses followed by 20mmol infusion |
IV 150-300mg, followed by an infusion of 900mg over 24 hrs |
Pharmacokinetics |
Rapid hepatic metabolism into inactive metabolites. |
Rapid distribution; some proportion becomes intracellular; the rest is renally excreted. |
Rapid distribution, with a vas volume of dsitribution; becomes bound to tissue proteins. |
Mechanism |
Inhibits voltage-gated sodium channels, decreasing the duration of action potentials and decreasing the duration of repolarisation |
Mechanism uncertain; appears to act as an antagonist to the entry of calcium into depolarising cells. |
Beta-blockade |
Adverse effects |
Neurological disturbances eg. paraesthesia, seizures |
Muscle weakness, decreased reflexes, hypotension |
Prolongation of QT interval, risk of Torsades. |
Compare and contrast the pharmacology of ceftriaxone, gentamicin and meropenem.
Ceftriaxone: vial with yellow water soluble powder for reconstitution; only administered parenterally, 33-66% excreted unchanged in urine, no active metabolites, 85-95% protein bound, elimination half life 6-9 hours (> 36 hours with severely impaired renal function), usual dosage 0.5 to 2g IV 12 or 24 hourly; 3rd generation cephalosporin antibiotic, inhibits cell wall synthesis, covers most gram negative rods (except Pseudomonas), and Gram positive cocci (except Methicillin Resistant, and group D streptococci); adverse reactions uncommon, but include overgrowth of non-susceptible organisms, and occasional haematologic, renal and hepatic adverse effects.
Gentamicin: ampoule with 80 mg/2 mL; only administered parenterally, excreted almost entirely by glomerular filtration, elimination half life 2-3 hours, no active metabolites, usual dosage 1 mg/kg tds or up to 5 mg/kg as daily dose, careful monitoring of blood levels required, especially if renal impairment (trough level not > 2 mcg/mL); aminoglycoside antibiotic, inhibits protein synthesis, covers most gram negative rods (including pseudomonas, but variability from hospital to hospital); serious adverse reactions include oto- and renal toxicity, potentiated by other oto- and nephro-toxins, prolongation of neuromuscular blockade may occur, other reactions uncommon.
Meropenem: vial with water soluble powder for reconstitution; only administered parenterally, 70% excreted unchanged in urine (requiring reduction of dosage if significant renal impairment), plasma binding 2%, elimination half life 1 hour, no active metabolites, usual dosage 500mg to 2g every 8 hours; carbapenem antibiotic, inhibits cell wall synthesis, active against a broad spectrum of aerobic and anaerobic bacteria (including Gram positive cocci and Gram negative rods, but excluding MRSA, Enterococcus faecium, Stenotrophomonas and many Pseudomonas); serious adverse reactions are rare, but include overgrowth of non- susceptible organisms, and occasional haematologic, gastrointestinal and hepatic adverse effects.
I refuse to believe that I would have lost marks in this question by not commenting on the colour of ceftriaxone powder.
Here is a tabulated answer with slightly less detail than the college provides.
|
Ceftriaxone |
Gentamicin |
Meropenem |
Class |
3rd gen cephalosporin |
Aminoglycoside |
Carbapenem |
Administration |
IV – 1g; up to 3g/day |
IV; daily dosing 5-7mg/kg |
IV 1-2g – up to 6g/day |
Pharmacokinetics |
Cleared both renally (unchanged) and via biliary excretion (33%) |
Cleared renally; rapidly and unchanged |
Cleared renally |
Mechanism of action |
Inhibition of bacterial cell wall synthesis by binding to penicllin-binding proteins |
Inhibition of bacterial protein synthesis by binding to the 30S subutnit of the bacterial ribosome |
Inhibition of bacterial cell wall synthesis by binding to penicllin-binding proteins |
Indications |
Infections by susceptible gram-positive organisms; |
Infections by susceptible gram-negative organisms; |
Infection by suceptible gram-positive and gram negative aerobic and anaerobic organisms; broad cover |
Precautions |
Seizures in overdose |
Nephrotoxicity |
Seizures in overdose |
Compare and contrast the clinical features and management of a patient following beta blocker overdose with those of a patient following calcium-channel blocker overdose.
Beta-blockers |
Ca channel blockers |
|
General |
Clinical features depend |
Varying CVS effects |
CVS |
Hypotension, |
Hypotension, bradycardia |
CCF |
||
Other systems |
Bronchospasm, |
Hyperglycemia, nausea |
Treatment |
Charcoal, fluids, Beta |
Charcoal, IV fluids, IV |
Though satisfactory, the college answer lacks qualities which help the studying candidate generate some memory of the differences between these drugs. The table below builds on the college answer by highlighting in bold the key differences between these overdoses.
|
Calcium channel blocker overdose |
Beta-blocker overdose |
Clinical features |
Bradycardia |
Bradycardia |
Management: |
||
Antidote |
Ionised calcium (eg. calcium chloride) |
Glucagon |
Decontamination |
Activated charcoal |
Activated charcoal |
Enhancement of clearance |
Hemoperfusion for verapimil |
Hemoperfusion for metoprolol |
DeWitt, Christopher R., and Javier C. Waksman. "Pharmacology, pathophysiology and management of calcium channel blocker and β-blocker toxicity." Toxicological reviews 23.4 (2004): 223-238.
a) List an antidote (1drug specific to the agent) in the event of an
overdose with each of the agents listed below in the table.
Benzodiazepines |
Beta blockers |
Cyanide |
Digoxin |
Heparin |
Iron |
Methanol, ethylene glycol |
Methaemoglobinemia |
Organophosphate |
Opiates |
Lead |
Paracetamol |
b) Which of the agents in the above list are not adsorbed by activated charcoal?
Benzodiazepines |
Flumazenil |
||
Beta blockers |
Glucagon, adrenaline |
||
Cyanide |
Na thiosulfate, hydroxocobalamin; |
||
Digoxin |
Fab, |
||
Heparin |
Protamine |
||
Iron |
Desferrioxamine |
||
Methanol, ethylene glycol |
ethanol |
||
Methaemoglobinemia |
Ascorbic acid, methylene blue |
||
Organophosphate |
Atropine, pralidoxime |
||
Opiates |
Naloxone |
||
Lead |
Dimercaprol |
||
Paracetamol |
N-Acetylcysteine |
||
b) Which of the agents in the above list are not adsorbed by activated charcoal?
Lead, alcohols, Fe, cyanide
This question closely resembles Question 28.1 from the second paper of 2009, as well as Question 14.2 from the first paper of 2008. It has slightly different drugs in its table, but otherwise it is essentially the same.
However, it does ask about the charcoal.
Thus:
The following drugs are NOT treatable by charcoal:
More on this can be found in a brief summary of ICU toxicology.
A 58 year old farmer with a history of depression was found collapsed in his shed. On arrival at the Emergency Department, his GCS was 10 (E2, V3, M5), respiratory rate was 23, and mouth ulceration was noted with a green coloured substance staining his lips, hands and clothes.
His arterial blood gas and biochemistry on admission were as follows:
FiO2 |
0.5 |
|
pH |
7.29 |
|
PCO2 (mmHg) |
35 (4.6 kPa) |
|
PaO2 |
68 (9.0 kPa) |
|
HCO3 (mmol/L) |
16 |
(24-28) |
Base Excess (mmol/L) |
-9 |
(-2.0 to |
Sodium (mmol/L) |
140 |
(135-145) |
Potassium (mmol/L) |
4.3 |
(3.5-5.0) |
Chloride (mmol/L) |
111 |
(95-105) |
Glucose (mmol/L) |
7.2 |
(4.0-6.0) |
Lactate (mmol/L) |
5.2 |
<2.5 |
Haemoglobin (g/L) |
162 |
(130-160) |
Creatinine |
230 |
(60-120) |
a. What is the likely diagnosis?
b. How can you confirm this?
c. List 4 important principles of management specific to this condition.
a. What is the likely diagnosis?
Paraquat ingestion
b. How can you confirm this?
Serum paraquat levels
History of exposure
c. List 4 important principles of management specific to this condition.
1) Risk assessment based on estimate of quantity of Paraquat ingested
2) Gastrointestinal decontamination with diatomaceous earths, activated charcoal or sodium resonium
3) Monitoring for organ dysfunction (respiratory, CVS, renal, GIT, adrenal, hepatic, CNS)
4) Avoid high FiO2
Though the most likely diagnosis is an overdose of some sort of horrible herbicide (and past history suggests the college likes their paraquat questions), one should still go though the motions of analysing a blood gas from basic principles.
Firstly, what we have here is a hypoxia with a widened A-a gradient.
The PAO2 should be (0.5 x 713) - (35 x 1.25), or 311mmHg - so the gradient is a whopping 246.
Next, we have a metabolic acidosis (the BE is -9)
This disorder is inadequately compensated by ventilation. No matter which equation you use, the CO2 should be lower. If you apply the "7.xx" rule, the CO2 shold be the last two digits of the pH - 29. If you apply Winter's Formula, the CO2 should be around 32. Thus, a mild respiratory acidosis also exists.
The anion gap is only slightly raised, 17.3 (140+4.3 - 111 - 16)
The delta ratio is therefore 0.66 (5.3 / 8) -if we take the normal anion gap to be 12.
The metabolic acidosis is therefore a mixed disorder.
The serum osmolality and urea are not provided, so we cannot calculate an osmolar gap.
Anyway... The gas exchange defect suggests pulmonary oedema, the bloods suggest renal failure, and the history screams herbicide. Paraquat selectively attacks the alveoli and causes renal necrosis. Ergo, its a case of paraquat poisoning. Another plausible explanation is early stages of ethylene glycol toxicity. Antifreeze is green - stained with fluoresceine so you can find radiator cracks more easily - and this could account for the hapless farmer's lips and hands. But it does not cause mucosal ulceration, and if the college really wanted the candidates to explore ethylene glycol as the main differential they would probably have provided them with a serum osmolality level.
Anyway. Diagnosis of paraquat toxicity consists of a suspicious history, confirmed by formal paraquat levels.
Management consists of supportive care of multi-organ system failure, and decontamination byFuller's Earth, which is essentially calcium montmorillonite, or bentonite - a absorbent aluminium phyllosilicate, formed from the weathering of volcanic ash.
Dialysis is probably going to be useless, as paraquat is rapidly eliminated and by the time you get the circuit set up most of it will have gone already. The alveolar and renal damage will have been done by then, so you have nothing to gain (other than a more rapid control of the acid-base disturbance).
Hyperoxia is to be avoided, as it has been demonstrated to exacerbate the oxidative toxicity of paraquat.
Gawarammana, Indika B., and Nicholas A. Buckley. "Medical management of paraquat ingestion." British journal of clinical pharmacology 72.5 (2011): 745-757.
Clark, D. G. "Inhibition of the absorption of paraquat from the gastrointestinal tract by adsorbents." British journal of industrial medicine 28.2 (1971): 186-188.
Kehrer, James P., Wanda M. Haschek, and Hanspeter Witschi. "The influence of hyperoxia on the acute toxicity of paraquat and diquat." Drug and chemical toxicology 2.4 (1979): 397-408.
Dinis-Oliveira, R. J., et al. "Paraquat poisonings: mechanisms of lung toxicity, clinical features, and treatment." Critical reviews in toxicology 38.1 (2008): 13-71.
List an antidote (1 drug specific to the agent) in the event of an overdose with each of the agents listed below in the table.
Agent |
Antidote |
Benzodiazepines |
|
Beta blockers |
|
Cyanide |
|
Digoxin |
|
Iron |
|
Methanol, Ethylene glycol |
|
Methemoglobinemia |
|
Organophosphate |
|
Opiates |
|
Paracetamol |
|
Agent |
Antidote |
Benzodiazepines |
Flumazenil |
Beta blockers |
Glucagon, adrenaline |
Cyanide |
Na thiosulfate, hydroxocobalamin, |
Digoxin |
Fab, |
Iron |
Desferrioxamine |
Methanol, Ethylene glycol |
Ethanol, 4 -methylpyruvate |
Methemoglobinemia |
Ascorbic acid, methylene blue |
Organophosphate |
Atropine, pralidoxime |
Opiates |
Naloxone |
Paracetamol |
N-Acetylcysteine |
This question closely resembles Question 28.1 from the second paper of 2009.
It has fewer drugs in its table, but otherwise it is essentially the same.
Compare the use of propofol and dexmedetomidine when used for sedation in the mechanically ventilated patient with specific reference to
A) pharmacodynamics
B) indications
C) complications
for each of the drugs. (You may tabulate your answer).
Propofol
Pharamcodynamics
• GABA receptor action, though different from benzodiazepine receptor
• Hydrophobic with high lipid solubility that allows it to cross blood brain barrier rapidly. Lipid solubility allows rapid redistribution to tissues so duration of action is only a few minutes.
B) Indications
• Sedation in ICU for ventilation
• Sedation for procedures such as ETT, endoscopy, TOE etc
• Sedation for transport
• Effective anticonvulsant.
C) Complications
• Cardiovascular: hypotension from preload reduction due to dilation of venous capacitance vessels & mild myocardial depression.
• Hyperlipidaemia possible: monitor triglyceride levels. Adjust TPN accordingly
• Propofol infusion syndrome: dysrhythmias, heart failure, metabolic acidosis, hyperkalaemia, rhabdomyolysis. Beware of high doses ( > 80 microg/kg/min) and/or higher concentrations ( 2% vs 1% ).
Dexmedetomidine
A)
Selective alpha-2 agonist with both sedative and analgesic properties.
B) Indications
Patients are sedated when undisturbed but they arouse easily with minimal stimulation, allowing frequent neurologic examinations. Useful in the agitated, ventilated patient.
• Analgesic sparing in post operative patients.
• Results in less delirium compared to benzodiazepines.
C) Complications
• Cardiovascular: bradycardia & hypotension. ( Vasoconstriction & hypertension have been reported with higher doses )
• Not well studied for long term administration to critically ill, mechanically ventilated patients.Licensed for use in Australia for 24 hours only, though utilised in trials for up to 120 hours.
Agian, the college has effectively and concisely summarised everything in their answer.
But, they didn't tabulate it, even though they invited the candidates to do so.
I have tabulated it for them.
Feature |
Propofol |
Dexmedetomidine |
Pharmacodynamics |
GABA agonist; |
Centrally acting alpha-2 receptor agonist; |
Indications |
Sedation (rapid onset) |
Sedation (slower in onset) |
Complications |
Loss of airway reflexes |
Bradycardia More adverse effects than with benzodiazepines |
Arain, Shahbaz R., and Thomas J. Ebert. "The efficacy, side effects, and recovery characteristics of dexmedetomidine versus propofol when used for intraoperative sedation." Anesthesia & Analgesia 95.2 (2002): 461-466.
Venn, R. M., and R. M. Grounds. "Comparison between dexmedetomidine and propofol for sedation in the intensive care unit: patient and clinician perceptions†."British journal of anaesthesia 87.5 (2001): 684-690.
Jakob, Stephan M., et al. "Dexmedetomidine vs midazolam or propofol for sedation during prolonged mechanical ventilation: two randomized controlled trials." JAMA307.11 (2012): 1151-1160.
Compare and contrast the pharmacology of dobutamine and levosimendan
LEVOSIMENDAN |
DOBUTAMINE |
|
Class |
Calcium sensitiser |
Synthetic catecholamine |
Pharmacokinetics |
Slower onset of action |
Onset of action in 1 – 2 min |
Mechanism of |
Increases myocyte sensitivity |
Selective beta-1 and beta-2 |
Therapeutic |
Increased cardiac |
Increased cardiac contractility |
LEVOSIMENDAN |
DOBUTAMINE |
|
Adverse effects |
Hypotension |
Drug-induced myocardial |
Contraindications |
Hypersensitivity to |
Hypersensitivity to dobutamine |
With such a detailed and comprehensive college answer, one can do nothing more than to rewrite the table with less information, so as to render the information it contains more memorable.
Features |
Dobutamine |
Levosimendan |
Class of drug |
Synthetic catecholamine |
Calcium sensitizer |
Administration |
IV infusion 5-15mcg/kg/min |
IV infusion 0.05-0.2mcg/kg/min |
Pharmacokinetics |
Rapidly metabolised by COMT; |
Excreted into the small intestine |
|
||
Mechanism of action |
Activates beta-1 adrencoeptors and increases heart rate and contractility by increasing the intracellular levels of cAMP, thus increasing the availablility of intracellular calcium. |
Enhances the affinity of contractile proteins (partiularly cardiac troponin C) for calcium, thereby increasing contractility without incurring additional ATP cost |
Clinical effects |
Increased inotoropy |
Increased inotropy |
Adverse effects |
Arrhythmia |
Arrhythmia |
Antila, Saila, Stig Sundberg, and Lasse A. Lehtonen. "Clinical pharmacology of levosimendan." Clinical pharmacokinetics 46.7 (2007): 535-552.
Papp, Zoltán, et al. "Pharmacological mechanisms contributing to the clinical efficacy of levosimendan." Cardiovascular drug reviews 23.1 (2005): 71-98.
With reference to intoxications, list the relevant physical features of hemodialysis and hemoperfusion filters which make them suitable for use and give one example of a toxin cleared by each of these.
The relevant physical features include
Haemoperfusion
Examples
Haemodialysis : Lithium, metformin
Haemoperfusion: Phenobarbitone, theophylline
A more detailed discussion is available regarding the use of haemoperfusion and haemodialysis in toxicology There's also a revision page about haemoperfusion in a broader context. Interestingly, the college question asked for the properties of filters which make them suitable for use, rather than the properties of the drugs (which is what the college answer consists of).
If were to actually answer the question, it would look something like this:
Haemodialysis filters:
Hemoperfusion filters:
Nenov, Vesselin D., et al. "Current applications of plasmapheresis in clinical toxicology." Nephrology dialysis transplantation 18.suppl 5 (2003): v56-v58.
Holubek, William J., et al. "Use of hemodialysis and hemoperfusion in poisoned patients." Kidney international 74.10 (2008): 1327-1334.
Ghannoum, Marc, et al. "Hemoperfusion for the treatment of poisoning: technology, determinants of poison clearance, and application in clinical practice." Seminars in dialysis. Vol. 27. No. 4. 2014.
Ghannoum, Marc, et al. "Blood purification in toxicology: nephrology’s ugly duckling." Advances in chronic kidney disease 18.3 (2011): 160-166.
Takki, S., et al. "Pharmacokinetic evaluation of hemodialysis in acute drug overdose." Journal of pharmacokinetics and biopharmaceutics 6.5 (1978): 427-442.
Define ideal body weight and what is its significance to dosing of drugs. Briefly, outline the effects morbid obesity (body mass index > 40kg/m2) may have on the pharmacokinetics of medications in critically ill adults.
° Ideal body weight is usually estimated from formulae or approximately:
IBW (kg) males =height cm -100, IBW (kg) females height cm -110
° Dosing weight is best worked out from ideal body weight.
Lean body weight or dosing weight = ideal body weight + (ABW-IBW) x 0.4
Pharmacokinetics
Distribution
° markedly affected by ratio of adipose tissue to lean body mass
° Increased volume of distribution for lipid soluble drugs
° Accumulation of lipophilic drugs in fat stores
° May increase dose needed to gain effect
° Vd of hydrophilic drugs less affected but blood, extracellular fluid, body organ, and connective tissue volume are also increased.
° Total body water may be increased by resuscitation volume etc
° Cmax reduced and T1/2 increased
° Lipid soluble drugs usually dosed on ABW, water soluble drugs dosed on ideal or lean body weight
Metabolism
° Variable effects. More likely to be affected by critical illness with drug interactions, reduced hepatic blood flow, altered protein binding
Excretion
° Obese patients with normal renal function have an increased glomerular filtration rate and thus an increased clearance of drugs excreted by the kidney. Co existing disease processes eg diabetes may change this
° Calculated and measured creatinine clearance correlate poorly in obesity and in critically ill
Thus morbidly obese predisposed to inadequate dosing and increased toxicity. Need to measure serum levels of drugs with low therapeutic index.
Ideal body weight:
Effect on absorption:
Effect on distribution:
Effect on metabolism:
Effect on clearance:
Effect on pragmatic drug dosing and monitoring:
De Baerdemaeker, Luc EC, Eric P. Mortier, and Michel MRF Struys. "Pharmacokinetics in obese patients." Continuing Education in Anaesthesia, Critical Care & Pain 4.5 (2004): 152-155.
Cheymol, Georges. "Effects of obesity on pharmacokinetics." Clinical pharmacokinetics 39.3 (2000): 215-231.
A two year old boy is suspected of ingesting iron tablets.
a) List three clinical signs of iron poisoning.
b) List two investigations which would support the diagnosis of iron poisoning.
c) Which blood gas (a or b or c) would be most consistent with iron poisoning? Justify your choice of answer.
a |
b |
c |
|
pH |
7.1 |
7.55 |
7.45 |
pCO2 |
34 mmHg (4.5 kPa) |
30 mmHg (4.5 kPa) |
34 mmHg (4.5 kPa) |
pO2 |
75 mmHg (10 kPa) |
90 mmHg (12 kPa) |
70 mmHg (9.3 kPa) |
BE |
-18 mmol/L |
+4 (mmol/L) |
-0.1 mmo/L |
d) List three treatments specific for iron poisoning and their mechanisms of action.
e) List one serious long term complication of iron poisoning.
a) List three clinical signs of iron poisoning.
Clinical sign |
Cause |
Nausea / vomiting/ haematemesis |
Acute gastritis, ischaemia |
Diarrhoea |
|
Abdominal pain |
|
Melaena |
|
Tachypnoea |
Metabolic acidosis |
Coma/ seizures |
|
Shock/ hypotension |
Myocardial depression |
Oliguria |
Capillary leak |
Jaundice / coagulopathy |
Hepatic necrosis |
b) List two investigations which would support the diagnosis of iron poisoning.
Iron Level > 300 microgm/dL, or 63 micromol/L |
|
Abdominal XR: |
Shows iron tablets |
Blood gas |
Metabolic acidosis |
Hyperglycaemia |
|
Coagulopathy |
Interference with coagulation cascade/ |
Deranged liver enzymes |
From hepatic necrosis |
Raised white cell count |
c) Which blood gas (a or b or c) would be most consistent with iron poisoning? Justify your choice of answer.
a |
b |
c |
|
pH |
7.1 |
7.55 |
7.45 |
pCO2 |
34 mmHg (4.5 kPa) |
30 mmHg (4.5 kPa) |
34 mmHg (4.5 kPa) |
pO2 |
75 mmHg (10 kPa) |
90 mmHg (12 kPa) |
70 mmHg (9.3 kPa) |
BE |
-18 mmol/L |
+4 (mmol/L) |
-0.1 mmo/L |
Answer: Metabolic acidosis due to uncoupling of oxidative phosphorylation.
d) List three treatments specific for iron poisoning and their mechanisms of action.
Desferrioxamine |
(Binds intravenous iron to form water soluble |
Whole bowel irrigation |
(Polyethylene glycol: works with minimal |
Exchange transfusion with plasmapheresis. |
|
Surgical/ endoscopic removal of tablets |
(If seen on AXR). |
Treat coagulopathy |
|
Treat hyperglycaemia |
|
Aggressive volume resuscitation (as |
|
Dialysis |
But limited efficacy |
Gastric Lavage with HCO3 |
(Controversial) |
Note: Charcoal is ineffective.
e) List one serious long term complication of iron poisoning.
1. Bowel obstruction (esp gastric outlet)
2. GI strictures
As this question closely resembles Question 8 from the second paper of 2013, I will not elaborate excessively.
a)
Feature | Causes |
Tachypnoea |
|
Shock, circulatory collapse |
|
Hypoglycaemia |
|
Coma |
|
High anion gap metabolic acidosis |
|
Hyperlactatemia |
|
Renal failure |
|
Gastric ulceration |
|
Haemorrhage, melaena |
|
b)
c)
d)
Decontamination
Enhanced elimination
Specific antidote
Supportive care
e)
Toxicity manifests in four stages, where the late Stage IV represents gastrointestinal scarring (4-6 weeks since ingestion) - gastric scarring and pyloric stricture are the specific features.
The Royal Childrens Hospital has a good set of guidelines for irone overdose.
Abhilash, Kundavaram PP, J. Jonathan Arul, and Divya Bala. "Fatal overdose of iron tablets in adults." Indian journal of critical care medicine: peer-reviewed, official publication of Indian Society of Critical Care Medicine 17.5 (2013): 311.
REISSMANN, KURT R., and THOMAS J. COLEMAN. "Acute Intestinal Iron Intoxication II. Metabolic, Respiratory and Circulatory Effects of Absorbed Iron Salts." Blood 10.1 (1955): 46-51.
REISSMANN, KURT R., et al. "Acute Intestinal Iron Intoxication I. Iron Absorption, Serum Iron and Autopsy Findings." Blood 10.1 (1955): 35-45.
Outline the role of vasopressin and its analogues in the critically ill patient.
• Vasopressin - Septic shock – Improves blood pressure, No evidence of mortality benefit- NEJM study, some benefit in the less sick population
• Terlipressin - Hepatorenal syndrome
• Diabetes insipidus – Desmopressin improves polyuria, restores serum Na concentrations
• Pitressin - Variceal bleed
• Vasopressin - Cardiopulmonary resuscitation
• Desmopressin - Post cardiopulmonary bypass bleeding – 20 units improves platetet dysfunction related bleed but may cause myocardial ischemia
• Von Willebrands’s
The college answer embarrasses itself by treating "pitressin" as something separate from vasopressin, whereas in fact they are the same damn thing.
Anyway. An extensive homage to vasopressin is available elsewhere. The discussion of its relatives seems to work better as a table.
Substance |
Physiology |
Uses in critical care |
Vasopressin |
|
Use in septic shock:
Use in cardiac arrest:
|
Terlipressin |
|
Use in hepatorenal syndrome
Use in variceal haemorrhage
Use in septic shock
|
Desmopressin |
|
Use in diabetes insipidus
Use in haemostasis
|
List an antidote (one (1) drug specific to the agent) in the event of an overdose with each of the agents listed below in the table.
Agent |
Antidote |
Benzodiazepines |
|
Beta blockers |
|
Bupivacaine |
|
Cyanide |
|
Digoxin |
|
Ethylene glycol |
|
Isoniazid |
|
Methanol, |
|
Methemoglobinemia |
|
Organophosphate |
|
Opiates |
|
Lead |
|
Valproate |
|
List an antidote (one (1) drug specific to the agent) in the event of an overdose with each of the agents listed below in the table.
Agent |
Antidote |
Benzodiazepines |
Flumazenil |
Beta blockers |
Glucagon |
Bupivacaine |
Intralipid |
Cyanide |
Cyanocbalamin/ Sodium thiosulphate |
Digoxin |
Fab |
Ethylene glycol |
Ethanol, Fomepizole |
Isoniazid |
Pyridoxine |
Methanol, |
Ethyl alcohol |
Methemoglobinemia |
Methylene blue |
Organophosphate |
Atropine |
Opiates |
Naloxone |
Lead |
Dimercaprol, BAL |
Valproate |
Carnitine |
This question does not warrant an especially extensive discussion.
Instead, I will link to intersting articles.
In the list provided by the college, there are standard drugs which everyone would know the antidotes for, and non-standard ones which may not be totally familiar to people without a toxicology background.
Pyridoxine is a co-factor in the synthesis of GABA; isoniazid interferes with this synthesis, and causes seizures in overdose. The supplementation of pyridoxine seems to prevent the worst of isoniazid toxicity (it seems the inhibition of lactate metabolism is not such a big deal).
Or so it is thought. The most disturbing aspects of valproate toxicity are valroate-induced hyperammonaemic encephalopathy and hepatotoxicity. Carnitine deficiency is implicated in both, and seems to be caused by chronic valproate administration more so than acute. The reason for the efficacy of carnitine in valproate overdose seems to stem from its central role in beta-oxidation of long chain fatty acids (which is the metabolic pathway taken by valproate). It appears to hasten the resolution of coma, and it seems to protect the liver from necrosis; the mechanism is thought to be the prevention of accumulation of toxic metabolites of valproate.
(Incidentally, carnitine is also being considered as a rescue therapy for propofol infusion syndrome)
And mercury, antimony, gold, chrome, cobalt and nickel poisoning. First developed to treat arsenic poisoning during the Second World War, dimercaprol (or British Anti-Lewisite, BAL) is a chelating agent which competes for heavy metal ions with the thiol groups of enzymes, thus preventing the inactivation of those enzymes. The metal-dimercaprol complex is then renally excreted.
Dimercaprol itself is horribly toxic, and its use in heavy metal poisoning is limited to situations where heavy metal levels are high, toxicity is already severe, and water-soluble analogues of dimercaprol (eg. DMPS and DMSA) are not available.
Murakami, K., et al. "Effect of L‐Carnitine Supplementation on Acute Valproate Intoxication." Epilepsia 37.7 (1996): 687-688.
Lheureux, Philippe ER, et al. "Science review: Carnitine in the treatment of valproic acid-induced toxicity–what is the evidence?." Critical care 9.5 (2005): 431.
Kam, P. C. A., and D. Cardone. "Propofol infusion syndrome." Anaesthesia 62.7 (2007): 690-701.
Peters, Rudolph A., Lloyd A. Stocken, and R. H. S. Thompson. "British anti-lewisite (BAL)." Nature 156.Nov. 24 (1945): 616.
There is an indepth entry on dimercaprol in www.inchem.org.
Briefly outline the mechanism of effectiveness of sodium bicarbonate in the management of tricyclic antidepressant overdose.
Increased serum pH, TCAs are weak bases and therefore increasing serum pH will increase the proportion of non-ionised drug thus causing a greater proportion of drug to be distributed throughout the body away from the heart.
Increased serum Na also overcomes the Na receptor blockade
Alkalinisation also accelerates recovery of sodium channels by neutralizing the protonation of the drug receptor complex.
The indication for the use of bicarbonate in tricyclic overdose is the widening of the QRS interval, rather than the metabolic acidosis (which may or may not accompany TCA poisoning).
Exactly how this works is a topic of some debate. In general, the QRS prolongation in TAC overdose seems to result from voltage-gated sodium channel blockade
Some authers have been able to demonstrate that amitryptilline enjoys greater protein binding in a more alkaline environment, which decreases the fraction of free drug.
Other authors have correctly identified sodium (rather than bicarbonate) as the more important ion in sodium bicarbonate; the administration of hypertonic saline seemed to have greater antiarrhytmic effect than sodium bicarbonate!
The last part of the college answer I could find no evidence for, at least not in the way it was worded. A good paper on the molecular mechanisms of sodium channel blockade by imipramine seems to report that intracellular alkalosis seems to favour the unbinding of imipramine from the voltage-gated sodium channel, which vaguely sounds like the thing that the college said.
In summary, bicarbonate in TCA overdose works in the following ways:
Hoffman, J. R., and C. R. McElroy. "Bicarbonate therapy for dysrhythmia and hypotension in tricyclic antidepressant overdose." Western Journal of Medicine134.1 (1981): 60.
Kerr, G. W., A. C. McGuffie, and S. Wilkie. "Tricyclic antidepressant overdose: a review." Emergency Medicine Journal 18.4 (2001): 236-241.
Brown, T. C., et al. "The use of sodium bicarbonate in the treatment of tricyclic antidepressant-induced arrhythmias." Anaesthesia and intensive care 1.3 (1973): 203-210.
McCabe, James L., et al. "Experimental tricyclic antidepressant toxicity: a randomized, controlled comparison of hypertonic saline solution, sodium bicarbonate, and hyperventilation." Annals of emergency medicine 32.3 (1998): 329-333.
Bou-Abboud, Elias, and Stanley Nattel. "Molecular mechanisms of the reversal of imipramine-induced sodium channel blockade by alkalinization in human cardiac myocytes." Cardiovascular research 38.2 (1998): 395-404.
(a) List the risk factors for and the clinical and laboratory findings of propofol infusion syndrome.
(b) Outline your management of a patient with suspected propofol infusion syndrome.
(a) List the risk factors for and the clinical and laboratory findings of propofol infusion syndrome.
Risk Factors
Large doses (> 4mg/kg/hr for > 48 hours in adults): typically, but not always, large dose, long time
Younger age
Acute neurological injury
Low carbohydrate intake
Catecholamine and/or corticosteroid infusion
Clinical and laboratory findings Unexplained lactic acidosis Increasing inotrope support
(Lipaemic serum, propofol levels / chromatography (if available??))
Brugada-like ECG abnormalities (Coved-type = convex-curved ST elevation in V1-
3)
(Green urine)
Cardiovascular collapse, reflected in PICCO / PAC / ECHO Rhabdomyolysis, high CK, hyperkalaemia
Arrhythmia / heart block
Renal failure
(b) Outline your management of a patient with suspected propofol infusion syndrome.
Management:
High index of suspicion
Discontinue immediately
Monitor for early warning signs: lactate, CK, Urine myoglobin, ECG Standard cardio-respiratory support
Consider pacing (bradycardia often resistant to high dose CA and pacing)
Adequate carbohydrate intake (6-8mg/kg/min)
Carnitine supplementation: theoretical benefit
Haemodialysis and haemoperfusion, used, unproven benefit
ECMO: 2 case reports, readily reversible pathology
Propofol infusion syndrome is discussed elsewhere.
It is well covered in an article by Prof Kam.
Pathophysiology of propofol infusion syndrome
a) Risk factors for propofol infusion syndrome
Clinical features and laboratory findings in propofol infusion syndrome
Management of propofol infusion syndrome
Enhanced elimination
Specific antidote
Supportive care
Kam, P. C. A., and D. Cardone. "Propofol infusion syndrome." Anaesthesia62.7 (2007): 690-701.
Marinella, Mark A. "Lactic acidosis associated with propofol." CHEST Journal109.1 (1996): 292-292.
Vasile, Beatrice, et al. "The pathophysiology of propofol infusion syndrome: a simple name for a complex syndrome." Intensive care medicine 29.9 (2003): 1417-1425.
Schenkman KA, Yan S. Propofol impairment of mitochondrial respiration in isolated perfused guinea pig hearts determined by reflectance spectroscopy. Critical Care Medicine 2000; 28: 172–7.
Fodale, Vincenzo, and Enza La Monaca. "Propofol Infusion Syndrome." Drug Safety 31.4 (2008): 293-303.
Da-Silva, Shonola S., et al. "Partial-exchange blood transfusion: an effective method for preventing mortality in a child with propofol infusion syndrome." Pediatrics 125.6 (2010): e1493-e1499.
Uezono, Shoichi, et al. "Acquired carnitine deficiency: a clinical model for propofol infusion syndrome?." The Journal of the American Society of Anesthesiologists 103.4 (2005): 909-909.
Mirrakhimov, Aibek E., et al. "Propofol Infusion Syndrome in Adults: A Clinical Update." Critical care research and practice 2015 (2015).
A 16 year old female is admitted to the ICU following a multiple drug overdose.
a)Outline the role of activated charcoal in the management of drug overdose.
b) What are the complications of activated charcoal therapy?
c) When is dialysis utilised in toxic syndromes?
d) In the context of an overdose, list 3 drugs for which charcoal haemoperfusion may be useful.
a) Outline the role of activated charcoal in the management of drug overdose.
• Single dose activated charcoal is generally preferred method of decontamination but does not improve outcome when applied to unselected patients and should not be regarded as routine.
• Indicated when likely that toxic agent is still within the GI tract (1st hour for most agents) and potential benefits outweigh risks.
b) What are the complications of activated charcoal therapy?
• Vomiting
• Pulmonary aspiration
• Direct administration to lung via misplaced NG tube (potentially fatal)
• Impaired absorption of oral medications / antidotes
• Corneal abrasions
• Constipation / bowel obstruction (MDAC)
c) When is dialysis utilised in toxic syndromes?
• Best if drug is:
• Water soluble
• MW <500
• Not highly protein bound
• Eg Lithium, Ethylene glycol, Salicylates, Na Valproate
• Also good for correcting fluid and electrolyte abnormalities
d) In the context of an overdose, list 3 drugs for which charcoal haemoperfusion may be useful.
• Common drugs carbamazepine, theophylline, paraquat
The rationale for multiple-dose charcoal is slightly different. It's not a matter of "just give more of it for more effect".
The following is a list of well-accepted indications for multiple dose activated charcoal (from Pierre Gaudrealt, 2005)
Drugs which are absorbed too rapidly
Drugs which do not adsorb on to charcoal
c)
Use of dialysis in toxicology:
d)
The website of the American Academ of Clinical Toxicology has several position statements which might be useful to the fellowship candidate:
Single-Dose Activated Charcoal
Gaudreault, Pierre. "Activated charcoal revisited." Clinical Pediatric Emergency Medicine 6.2 (2005): 76-80.
Andersen, A. Harrestrup. "Experimental Studies on the Pharmacology of Activated Charcoal. III. Adsorption from Gastro‐Intestinal Contents." Acta Pharmacologica et Toxicologica 4.3‐4 (1948): 275-284.
Krenzelok, Edward P. "New developments in the therapy of intoxications." Toxicology letters 127.1 (2002): 299-305.
Eddleston, Michael, et al. "Multiple-dose activated charcoal in acute self-poisoning: a randomised controlled trial." The Lancet 371.9612 (2008): 579-587.
Isbister, Geoffrey K., and Venkata V. Pavan Kumar. "Indications for single-dose activated charcoal administration in acute overdose." Current opinion in critical care 17.4 (2011): 351-357.
Chyka, P. A., and D. Seger. "Position statement: single-dose activated charcoal. American Academy of Clinical Toxicology; European Association of Poisons Centres and Clinical Toxicologists." Journal of toxicology. Clinical toxicology 35.7 (1996): 721-741.
Harris, Carson R., and Dean Filandrinos. "Accidental administration of activated charcoal into the lung: aspiration by proxy." Annals of emergency medicine22.9 (1993): 1470-1473.
Chyka, P. A., and D. Seger. "Position statement: single-dose activated charcoal. American Academy of Clinical Toxicology; European Association of Poisons Centres and Clinical Toxicologists." Journal of toxicology. Clinical toxicology 35.7 (1996): 721-741.
Isbister, Geoffrey K., and Venkata V. Pavan Kumar. "Indications for single-dose activated charcoal administration in acute overdose." Current opinion in critical care 17.4 (2011): 351-357.
Harris, Carson R., and Dean Filandrinos. "Accidental administration of activated charcoal into the lung: aspiration by proxy." Annals of emergency medicine22.9 (1993): 1470-1473.
UpToDate has a nice table of drugs which are removed by haemoperfusion.
Nenov, Vesselin D., et al. "Current applications of plasmapheresis in clinical toxicology." Nephrology dialysis transplantation 18.suppl 5 (2003): v56-v58.
Holubek, William J., et al. "Use of hemodialysis and hemoperfusion in poisoned patients." Kidney international 74.10 (2008): 1327-1334.
Ghannoum, Marc, et al. "Hemoperfusion for the treatment of poisoning: technology, determinants of poison clearance, and application in clinical practice." Seminars in dialysis. Vol. 27. No. 4. 2014.
Ghannoum, Marc, et al. "Blood purification in toxicology: nephrology’s ugly duckling." Advances in chronic kidney disease 18.3 (2011): 160-166.
Takki, S., et al. "Pharmacokinetic evaluation of hemodialysis in acute drug overdose." Journal of pharmacokinetics and biopharmaceutics 6.5 (1978): 427-442.
a) Outline the effect of critical illness on enteral drug absorption
b) List the reasons for altered drug clearance in the critically ill.
a) Outline the effect of critical illness on enteral drug absorption
• Multiple factors may alter gastrointestinal mucosal absorption including mucosal oedema, disordered gastrointestinal motility and disordered mucosal blood flow
• Gastric emptying / gut motility affected by drugs (opioids. Anticholinergics, antacids, inotropes), enteral nutrition, brain or spinal injury, diabetes
• Incomplete oral medication disintegration or dissolution
• Changes in pH
b) List the reasons for altered drug clearance in the critically ill.
Liver function
Reduced clearance
With hepatic dysfunction present in more than half the critically ill patients, drug clearance may be reduced because of :
a. Lower hepatic blood flow
b. Decreased hepatocellular enzyme activity c. Lower bile flow
d. Administration of other drugs competing for enzymes
Increased clearance
Hepatic enzyme induction by certain drugs may increase clearance of others
Renal function
Reduced clearance
Compromised kidney function may be secondary to reduced perfusion, intrinsic damage secondary to ischaemia or drug toxicity and immunologic injury
A decrease in GFR would increase the half-life of medications that are renally cleared and may result in drug or metabolite accumulation
Increased clearance
Increased cardiac output in early sepsis increases GFR and increased drug clearance Burns, use of diuretics and hypertonic saline also result in increased GFR and potentially increase clearance
Protein binding changes
Three major proteins affecting drug protein binding – albumin, alpha 1 acid glycoprotein and lipoproteins
Reduced clearance
Some proteins (eg alpha 1-acid glycoprotein binding morphine) are increased in critically ill resulting in reduced clearance
Increased clearance
Albumin is reduced so there will be a higher concentration of free drug for drugs normally bound to albumin resulting in increased clearance
Protein binding affected by other factors including accumulation of endogenous binding
inhibitors, qualitative changes on binding sites, competition for binding by other substances, pH changes
a) in short, critical illness decreases drug absorption by the following mechanisms:
b) critical illness may reduce drug clearance by the following mechanisms:
Boucher, Bradley A., G. Christopher Wood, and Joseph M. Swanson. "Pharmacokinetic changes in critical illness." Critical care clinics 22.2 (2006): 255-271.
A 25-year-old man presents to the Emergency Department following suspected snake bite. He has an effective pressure-immobilisation bandage in situ.
Indications for the use of polyvalent antivenom in snake envenomation:
c) Role of pharmacological pretreatment prior to the administration of snake antivenom:
d) Parameters:
Several possibilities here and many controversies:
Investigations for a snake bite victim:
Indications for polyvalent antidote:
Evidence for premedication for antivenom administration:
How do you know your monovalent antivenom is working?
Isbister, Geoffrey K., et al. "Snakebite in Australia: A practical approach to diagnosis and treatment." Medical journal of Australia 199.11 (2013): 763-768.
A 45-year-old man is admitted to the Emergency Department after ingesting an unknown quantity of “headache tablets”. His initial complaints are nausea, vomiting, shortness of breath and tinnitus. Fluid resuscitation has been commenced. You are asked to assess him as he is getting more dyspnoeic.
His serum biochemistry and arterial blood gas profile are as follows:
Parameter |
Result |
Normal Range |
Sodium |
138 mmol/L |
135 – 145 |
Potassium |
3.2 mmol/L* |
3.4 – 5.0 |
Chloride |
108 mmol/L |
100 – 110 |
Bicarbonate |
10 mmol/L* |
22 – 27 |
FiO2 |
0.3 |
|
pH |
7.32* |
7.35 – 7.45 |
PO2 |
125 mmHg (16.4 kPa) |
|
PCO2 |
20 mmHg (2.6 kPa)* |
35 – 45 (4.6 – 6.0) |
Base Excess |
-10 mmol/L* |
-2 – +2 |
Salicylate level |
105 mg/dL* |
3 – 10 mg/dL |
Paracetamol level |
<20 mg/L (<130 µmol/L) |
<20 (<130 µmol/L) |
a)
Acid-base status:
b)
c)
Hypoprothrombinaemia
Vitamin K
d)
Forced alkaline diuresis. Renal excretion of salicylates becomes important when the metabolic pathways become saturated. There is a 10-20 fold increase in elimination when the urine pH increased from 5 to 8
Haemodialysis. Most of the drug is protein-bound, and is concentration dependant. The volume of distribution is small, and binding site saturation leads to large levels of free drug, which is easily dialysable
Multiple-dose charcoal. Many aspirin forms are slow release and after ingestion they clump together in the GI tract, forming a large slow release preparation. It is also poorly soluble in the stomach leading to delayed absorption.
a)
The change in anion gap is 10, and the drop in bicarbonate is 14, which gives a delta ratio of 0.8, suggesting that there is a mixed high anion gap and normal anion gap metabolic acidosis.
There is indeed a respiratory alkalosis, which is appropriate (the rules of compensation suggest that the CO2 should be about 23).
b)
Salicylate toxicity has a whole list of complications. The college had asked specifically for severe ones. One may conceive of a respiratory alkalosis so dramatic as to warrant this adjective, and the same can be said for just about any other complication of salicylate toxicity, so they are all listed here.
Serum level 30-50mg/dL: | Serum level 50-75mg/dL: | Serum level >75mg/dL: |
|
|
|
c)
It is known that salicylate toxicity can cause a decrease in prothrombin.
Vitamin K (if not prothrombinex) is the answer.
d)
Severe toxicity from salicylates has several treatment options:
Decontamination
Direct and indirect antidotes
Enhancement of clearance
Supportive ICU therapies
O'Malley, Gerald F. "Emergency department management of the salicylate-poisoned patient." Emergency medicine clinics of North America 25.2 (2007): 333-346.
Pinedo, H. M., L. B. van de Putte, and E. A. Loeliger. "Salicylate-induced consumption coagulopathy." Annals of the rheumatic diseases 32.1 (1973): 66.
Shapiro, Shepard, Milton H. Redish, and Harold A. Campbell. "Studies on Prothrombin: IV. The Prothrombinopenic Effect of Salicylate in Man."Experimental Biology and Medicine 53.2 (1943): 251-254.
Pearlman, Brian L., and Rashi Gambhir. "Salicylate Intoxication." Postgraduate medicine 121.4 (2009).
The following is an image from an abdominal CT scan taken of a 24 year old man who presented with a carbamazepine overdose.
What complication has occurred?
Gastrointestinal obstruction secondary to multi dose charcoal administration.
It is surprisingly difficult to find a CT scan of a charcoal bezoar. One might think that carbamazepine+charcoal+"CT abdo" would be a specific enough search string to find the exact image in the ind of the examiner, as the scenario described here simpoly screams "case report", and in fact that is exactly what you get; except the case report was published seven years after this SAQ came out. Aljohani et al (2019) describe a 22-year-old patient who had received multiple-dose activated charcoal for carbamazepine intoxication. The CT, shown above, demonstrated "small bowel obstruction to the level of the proximal ileal loops, with a transition point between the dilated proximal loops and the collapsed terminal ileal loops"
Aljohani, Turki Khaled, et al. "A rare case of small bowel obstruction secondary to activated charcoal administration." Journal of surgical case reports 2019.2 (2019): rjz033.
Watson, William A., Karl F. Cremer, and James A. Chapman. "Gastrointestinal obstruction associated with multiple-dose activated charcoal." The Journal of emergency medicine 4.5 (1986): 401-407.
Goulbourne, Karita Boyd, and James E. Cisek. "Small-bowel obstruction secondary to activated charcoal and adhesions." Annals of emergency medicine 24.1 (1994): 108-110.
Chan, Justin CY, Chaminda Saranasuriya, and Bruce P. Waxman. "Bezoar causing small bowel obstruction after repeated activated charcoal administration." Medical Journal of Australia 183.10 (2005): 537.
What key cardiac effects are observed with acute digoxin toxicity? List two rhythm disturbances highly associated.
List three drugs known to enhance digoxin serum level. Provide a mechanism for each.
Other than drugs, what other factors are known to exacerbate digoxin toxicity?
With respect to the use of digoxin specific Fab fragments:
a)
Key cardiac features are increased automaticity combined with AV conduction block.
Rhythms suggestive: PAT with variable block
Accelerated junctional rhythms
Bidirectional ventricular tachycardia (specific for Digoxin).
Other (a variety are seen): SA node arrest, premature ventricular contractions, bradycardia, non paroxysmal junctional tachycardia, AV nodal blockade, ventricular tachycardia, ventricular flutter and fibrillation.
Note: Features of digoxin effect (e.g. T wave flattening/ inversion) do not correlate well with toxicity.
b)
Verapamil, Diltiazem, Amiodarone via inhibition of P-glycoprotein (efflux pump that excretes many drugs, including Digoxin, into the intestine or proximal renal tubule) - effectively reducing renal and GI secretion.
Erythromycin, omeprazole via increased Digoxin absorption.
c)
Low potassium, magnesium, pH, high calcium.
d)
Early recognition of toxicity and prompt administration of Fab fragments essential for severe poisoning. The serum Digoxin concentration does not necessarily correlate with toxicity.
Indications Include:
Life threatening arrhythmia with cardiovascular instability
Evidence of end organ dysfunction
Hyperkalaemia (> 5.0 – 5.5 mEq/l)
Ingestion of 10mg or more in total
After Fab administration free Digoxin levels are decreased to zero within minutes. Total Digoxin level will increase markedly since assays measure bound and free. Bound fraction rises due to an increase in Digoxin-Fab complex. These high levels have no correlation with toxicity and the serum level may be unreliable for several days and no action should be taken based on total level after digoxin-specific Fab fragments administration.
a)
The features of digoxin toxicity can be divided into cardiac and non-cardiac.
b)
Drug interactions of digoxin are a massive topic. The ones which result in overdose can be divided into inhibition of clearance (by inhibition of P-glycoprotein ) and increase of absorption.
c)
Digoxin toxicity is exacerbated by the following factors:
d)
Indications for the use of digoxin-specific Fab fragments are strange.
Life-threatening arrhythmia, hyperkalemia and altered mental status are mentioned, but the article in UpToDate recommends that digoxin antibodies be used in every poisoning, because there is no therapy with a comparable efficacy and safety.
"Total serum digoxin level continues to remain high after the administration of an appropriate dose of digoxin specific Fab fragments. What action would you take and why? "
One appropriate action would be to do nothing. The digoxin assay measures the total digoxin, whereas the free digoxin level after Fab may in fact be reduced to nearly zero. One is then confronted with a situation where the measured digoxin level is still very high, but the patient looks perfectly fine.
In such a situation, one should ignore the total level. I thank Yun from Canberra for pointing out the error in my initial reading of this question. If the clinical features of toxicity have resolved, the total digoxin level is meaningless. If they have not resolved, the patient requires another dose of the specific Fab fragments. If for whatever reason this is inadewuate, one may attempt resin hemoperfusion. However, this is not universally acknowledged as a useful strategy. Fab fragments together with plasmapheresis is another experimental technique.
UpToDate has a nice article.
Hauptman, Paul J., and Ralph A. Kelly. "Digitalis." Circulation 99.9 (1999): 1265-1270.
Marcus, Frank I. "Pharmacokinetic interactions between digoxin and other drugs." Journal of the American College of Cardiology 5.5s1 (1985): 82A-90A.
Gabello, M., et al. "Omeprazole induces gastric permeability to digoxin."Digestive diseases and sciences 55.5 (2010): 1255-1263.
Juneja, Deven, et al. "Severe suicidal digoxin toxicity managed with resin hemoperfusion: A case report." Indian journal of critical care medicine: peer-reviewed, official publication of Indian Society of Critical Care Medicine 16.4 (2012): 231.
Hauptman, Paul J., and Ralph A. Kelly. "Digitalis." Circulation 99.9 (1999): 1265-1270.
List the pharmacological characteristics of phenytoin and outline how these impacts on its clinical use.
Pharmacology |
Impact on clinical use |
Formulation: oral tablets or suspension |
Administration: oral or slow intravenous injection |
Good oral bioavailability (85%). Rate |
Enteral and intravenous doses are the same |
Distribution: approximately 90% |
Free drug concentration may be therapeutic in |
Elimination predominantly hepatic |
Several days to reach a new steady state and |
Kinetics: first order at low |
Changes in maintenance doses should be small. |
Mode of action: |
Clinical use as antiepileptic drug predominantly |
Hepatic enzyme inducer |
Increased metabolism and lower drug |
Other interactions |
Phenytoin concentrations may be increased by |
Adverse effects: |
Decrease rate of infusion, monitor blood pressure |
Properties | Influence on ICU management | |
Chemical properties |
|
|
Chemical relatives |
|
|
Administration |
|
|
Absoption |
|
|
Distribution |
|
|
Protein binding |
|
|
Metabolism/elimination |
|
|
Half life |
|
|
Mechanism of action |
|
|
Indications |
|
|
Contraindications |
|
|
Adverse effects |
|
|
Interactions |
|
Common ICU drugs which interfere with phenytoin:
|
Acute overdose |
|
|
A two-year-old boy is suspected of ingesting iron tablets.
a)List the clinical features, and the underlying pathophysiology, of iron poisoning.
b) Briefly outline your management of this child.
a)
Clinical Feature |
Mechanism |
|
Nausea, vomiting, diarrhoea |
Direct corrosive effect on GIT |
|
Abdominal pain |
Direct corrosive effect on GIT |
|
Gut ischaemia |
||
Disruption of cellular metabolism |
||
Shock |
Fluid losses from GIT |
|
3rd space losses and vasodilatation |
||
Anion gap metabolic acidosis |
Disruption of cellular metabolism |
|
Acute liver failure |
Disruption of cellular metabolism |
|
Shock and hypovolaemia |
||
Jaundice, coma, low BSL, coagulopathy |
||
Hepatic necrosis |
||
Renal failure |
Disruption of cellular metabolism |
|
Oliguria |
Shock and hypovolaemia |
b)
The pediatric aspect of this question does not feature prominently in the answer. The only time it is mentioned is in the discussion of whole bowel irrigation, and how foolish it would be to subject a two-year old to this.
a) is well presented by the college.
A flowchart of the mechanisms of high anion gap metabolic acidosis due to iron poisoning is presented elsewhere.
I will reproduce it here, for convenience.
Feature | Causes |
Tachypnoea |
|
Shock, circulatory collapse |
|
Hypoglycaemia |
|
Coma |
|
High anion gap metabolic acidosis |
|
Hyperlactatemia |
|
Renal failure |
|
Gastric ulceration |
|
Haemorrhage, melaena |
|
Toxicity manifests in four stages:
b) A systematic approach to an answer would resemble the following:
Decontamination
Enhanced elimination
Specific antidote
Supportive care
The Royal Childrens Hospital has a good set of guidelines for irone overdose.
Abhilash, Kundavaram PP, J. Jonathan Arul, and Divya Bala. "Fatal overdose of iron tablets in adults." Indian journal of critical care medicine: peer-reviewed, official publication of Indian Society of Critical Care Medicine 17.5 (2013): 311.
REISSMANN, KURT R., and THOMAS J. COLEMAN. "Acute Intestinal Iron Intoxication II. Metabolic, Respiratory and Circulatory Effects of Absorbed Iron Salts." Blood 10.1 (1955): 46-51.
REISSMANN, KURT R., et al. "Acute Intestinal Iron Intoxication I. Iron Absorption, Serum Iron and Autopsy Findings." Blood 10.1 (1955): 35-45.
A 62-year-old female is brought into hospital with suspected organophosphate poisoning.
a) List six acute clinical features associated with this condition.
b) List the antidotes indicated in this condition and the rationale for their use.
The following data are taken from this patient:
Parameter |
Patient Value |
Normal Adult Range |
Cholinesterase |
0.3 KU/L* |
3.4 – 9.0 |
Cholinesterase mixing |
33%* |
100% |
c) What does the result of the mixing test indicate?
a)
b)
c)
The first part of the question asks the candidate to produce 6 features of the cholinergic toxidrome. This should be a piece of cake. One recalles the mnemonic SLUDGEM:
The college answer does not lend itself well to being so easily memorised, and has broken at least one anagram engine. However, Yun from Canberra has pointed out that it is taken directly from the Australian Toxicology Handbook. The first six points are DUMBBELS (the muscarinic features), and the rest are nicotinic.
b)
Atropine and pralidoxime were asked for. The brevity of the college answer cannot be improved upon.
c)
In the mixing test, the patients serum and some random reference serum are both tested for plasma cholinesterase, and then a 50-50 mixture of the two is tested.
If there is enough pralidoxime being given, there will be little free organophosphate in the patient's sample, and the mixed sample will have a plasma cholinesterase level which is exactly between the patients sample and the reference sample.
If there is still free organophosphate present, then it will disable the plasma cholinesterase in the reference sample, and the cholinesterase level of the mixed sample will be surprisingly low.
Brian Kloss from LITFL has a superb cartoon to illustrate the horrors of the cholinergic toxidrome.
Sungur, Murat, and Muhammed Güven. "Intensive care management of organophosphate insecticide poisoning." Critical care 5.4 (2001): 211.
Kamanyire, R., and L. Karalliedde. "Organophosphate toxicity and occupational exposure." Occupational Medicine 54.2 (2004): 69-75.
Jr, Bailus Walker, and Joseph Nidiry. "Current concepts: organophosphate toxicity." Inhalation toxicology 14.9 (2002): 975-990.
With respect to the management of a 35-year-old female presenting with toxicity secondary to deliberate self-harm with paracetamol:
a) Outline how paracetamol causes liver dysfunction and how N-acetylcysteine (NAC) works as an antidote in this setting.
b) List the criteria for liver transplantation in this patient.
c) Outline your management of the patient in the event of clinical deterioration, pending transfer to the regional centre for liver transplantation.
a)
Paracetamol is predominantly conjugated into glucuronate and sulphate moeities
Small percentage is metabolized by cytochrome P450 to a toxic metabolite NAPQI, N-acetyl-p-benzoquinone imine (also known as NABQI).
Amount of NAPQI will vary according to genetic profile.
NAPQI is conjugated with glutathione to non-toxic moieties.
In paracetamol toxic ingestion the phase 2 conjugation enzymes are saturated so a higher fraction is converted to the toxic metabolite.
Conjugation of NAPQI with glutathione continues until it is depleted.
Toxic NAPQI accumulates and causes direct damage to hepatocytes.
NAC is a glutathione surrogate that detoxifies the toxic metabolite of paracetamol
NAC is converted to glutathione increasing the sulphation of paracetamol which prevents formation of the toxic metabolite blunting the localised inflammatory response in the liver.
b)
Arterial pH < 7.3 or lactate > 3.0 mmol/L after adequate resuscitation
OR
If all 3 of the following occur within a 24 hour period
Creatinine > 300 μmol/L
PT >100 seconds (INR > 6.5)
Grade 3 – 4 encephalopathy
c)
General supportive care with specific liver supportive therapy
Continue NAC
Ventilate as required
Normocarbia
Support the circulation
Fluids cautiously to avoid worsening cerebral oedema
Catecholamines / vasopressors
Early CRRT for renal failure
Control of acidaemia
Control of fluid balance
Avoid fever
Commence nutrition involving liver specific feeds with low amino acids
Lactulose 30 mL mg tds with other aperients
Thiamine loading large dose 300 mg iv tds
Regular vitamin K 10 mg iv daily
Avoid FFP unless requiring coverage for invasive procedures
Surveillance for infection and early antibiotic therapy if required
Stress-ulcer and DVT prophylaxis
Avoid hypoglycaemia
Control ICP
Examiners' comments: Candidates who did not pass gave sparse answers without sufficient detail, e.g. answer to part (b) was given as "King's College criteria" without further explanation.
a)
Mechanism of paracetamol toxicity is discussed elsewhere. Special attention is also given to the mitochondrial toxicity of paracetamol, which gives rise to lactic acidosis. In brif:
b)
The examiners complained that the trainees merely mentioned the King's College criteria by name. The model answer lists the actual criteria, implying that the trainees are expected to memorise them. The whole issue of prognostication in acute liver failure is discussed elsewhere, and the abovementioned criteria are only one of the possible ways of prognosticating - presumably, somebody who mentioned the MELD criteria would have also received a few marks.
King's College (O'Grady) Criteria - for paracetamol overdose
King's College (O'Grady) Criteria - for NON-paracetamol acute liver failure
c)
Management plans should include the following points:
O’Grady, John G., et al. "Early indicators of prognosis in fulminant hepatic failure." Gastroenterology 97.2 (1989): 439-445.
Daly, Frank FS, et al. "Guidelines for the management of paracetamol poisoning in Australia and New Zealand-explanation and elaboration." Medical journal of Australia 188.5 (2008): 296.
Dhiman, Radha K., et al. "Early indicators of prognosis in fulminant hepatic failure: An assessment of the Model for End‐Stage Liver Disease (MELD) and King's College Hospital Criteria." Liver transplantation 13.6 (2007): 814-821.
Yantorno, Silvina E., et al. "MELD is superior to King's college and Clichy's criteria to assess prognosis in fulminant hepatic failure." Liver transplantation13.6 (2007): 822-828.
A 58-year-old farmer with a history of depression was found collapsed in his shed. On arrival at the Emergency Department, his Glasgow Coma Scale score was 10 (E2, V3, M5), respiratory rate was 23 breaths per minute, and mouth ulceration was noted with a green coloured substance staining his lips, hands and clothes.
His arterial blood gas and biochemistry on admission were as follows:
Parameter | Patient Value | Normal Adult Range |
FiO2 | 0.5 | |
pH | 7.29* | 7.35 – 7.45 |
PCO2 | 35 mmHg (4.6 kPa) | 35 – 45 (4.6 – 6.0) |
PaO2 | 68 mmHg | (9.0 kPa) |
HCO3 | 16 mmol/L* | 24 – 28 |
Base Excess | -9.0 mmol/L* | -2.0 – +2.0 |
Sodium | 140 mmol/L | 135 – 145 |
Potassium | 4.3 mmol/L | 3.5 – 5.0 |
Chloride | 111 mmol/L* | 95 – 105 |
Glucose | 7.2 mmol/L* | 4.0 – 6.0 |
Lactate | 5.2 mmo/L* | < 2.5 |
Haemoglobin | 162 g/L* | 130 – 160 |
Creatinine | 230 µmol/L* | 60 – 120 |
a) Characterise the acid-base and blood gas abnormalities.
b) What is the likely diagnosis?
c) List the important principles of management specific to this condition.
a) Characterise the acid-base and blood gas abnormalities.
Combined high anion gap and normal anion gap metabolic acidosis with inadequate respiratory compensation (respiratory acidosis)
A-aDO2 = 245
b) What is the likely diagnosis?
Paraquat ingestion
c) List the important principles of management specific to this condition.
Risk assessment based on estimate of quantity of Paraquat ingested
Gastrointestinal decontamination with diatomaceous earths, activated charcoal or sodium resonium
Monitoring for organ dysfunction (respiratory, CVS, renal, GIT, adrenal, hepatic, CNS)
Avoid high FiO2
This data set is identical to that of Question 14.1 from the first paper of 2008.
The findings suggest paraquat toxicity:
Mild overdose:
|
Moderate overdose:
|
Massive overdose:
|
The toxicity (at least in moderate doses) emerges in several discrete phases:
Management of paraquat overdose follows the following pattern:
Decontamination
Enhancement of elimination
Specific antidotes
Supportive management
Gawarammana, Indika B., and Nicholas A. Buckley. "Medical management of paraquat ingestion." British journal of clinical pharmacology 72.5 (2011): 745-757.
Clark, D. G. "Inhibition of the absorption of paraquat from the gastrointestinal tract by adsorbents." British journal of industrial medicine 28.2 (1971): 186-188.
Kehrer, James P., Wanda M. Haschek, and Hanspeter Witschi. "The influence of hyperoxia on the acute toxicity of paraquat and diquat." Drug and chemical toxicology 2.4 (1979): 397-408.
Dinis-Oliveira, R. J., et al. "Paraquat poisonings: mechanisms of lung toxicity, clinical features, and treatment." Critical reviews in toxicology 38.1 (2008): 13-71.
Sirker, A. A., et al. "Acid− base physiology: the ‘traditional’and the ‘modern’approaches." Anaesthes
The following data refer to a 28-year-old male who is day 5 in ICU following a severe traumatic
brain injury. He has no other injuries and has been heavily sedated with infusions of fentanyl, midazolam and propofol since admission. Over the last four hours he has become increasingly bradycardic and hypotensive, and has not responded to fluid loading or repeated doses of atropine.
Parameter | Patient Value | Normal Adult Range |
Sodium | 138 mmol/L | 135 – 145 |
Potassium | 5.1 mmol/L* | 3.5 – 4.5 |
Chloride | 100 mmol/L | 95 – 105 |
Bicarbonate | 11 mmol/L* | 22 – 26 |
Urea | 29 mmol/L* | 2.9 – 8.2 |
Creatinine | 310 μmol/L* | 70 – 120 |
Calcium (corrected) | 1.71 mmol/L* | 2.10 – 2.55 |
Phosphate | 2.31 mmol/L* | 0.65 – 1.45 |
Creatine Kinase | 25,000 U/L* | 0 – 270 |
Lactate | 5.1 mmol/L* | < 2.0 |
Give the most likely diagnosis and the rationale for your answer.
(20% marks)
Propofol Infusion Syndrome.
Rationale: Biochemistry consistent with rhabdomyolysis. No other injuries to account for
it. Refractory bradycardia and hypotensive suggestive. History of high dose propofol
administration.
(Partial credit given for rhabdomyolysis, raised ICP and coning)
Propofol infusion syndrome is not the first thing I would think of when confronted with a trauma patient who has suddenly become hypotensive and bradycardic. And the raised CK does not help (as if rhabdomyolysis is unknown in trauma patients). However, the college threw in the "refractory to atropine" thing, which arouses concern.
Propofol infusion syndrome is discussed elsewhere.
It is well covered in an article by Prof Kam.
Clinical features of propofol infusion syndrome
Kam, P. C. A., and D. Cardone. "Propofol infusion syndrome." Anaesthesia62.7 (2007): 690-701.
Marinella, Mark A. "Lactic acidosis associated with propofol." CHEST Journal109.1 (1996): 292-292.
Vasile, Beatrice, et al. "The pathophysiology of propofol infusion syndrome: a simple name for a complex syndrome." Intensive care medicine 29.9 (2003): 1417-1425.
Schenkman KA, Yan S. Propofol impairment of mitochondrial respiration in isolated perfused guinea pig hearts determined by reflectance spectroscopy. Critical Care Medicine 2000; 28: 172–7.
Fodale, Vincenzo, and Enza La Monaca. "Propofol Infusion Syndrome." Drug Safety 31.4 (2008): 293-303.
Da-Silva, Shonola S., et al. "Partial-exchange blood transfusion: an effective method for preventing mortality in a child with propofol infusion syndrome." Pediatrics 125.6 (2010): e1493-e1499.
Uezono, Shoichi, et al. "Acquired carnitine deficiency: a clinical model for propofol infusion syndrome?." The Journal of the American Society of Anesthesiologists 103.4 (2005): 909-909.
Mirrakhimov, Aibek E., et al. "Propofol Infusion Syndrome in Adults: A Clinical Update." Critical care research and practice 2015 (2015).
A 46-year-old male from a foreign fishing vessel presents unconscious to the Emergency Department. He complained of visual disturbance prior to his deterioration.
The following blood results are obtained:
Parameter | Patient Value | Normal Adult Range |
Sodium | 144 mmol/L | 135 – 145 |
Potassium | 4.0 mmol/L | 3.5 – 5.0 |
Chloride | 102 mmol/L | 95 – 110 |
Bicarbonate | 8.2 mmol/L* | 22.0 – 30.0 |
Urea | 6.4 mmol/L | 3.0 – 7.0 |
Creatinine | 127 μmol/L* | 44 – 97 |
Glucose | 5.0 mmol/L | 3.5 – 7.8 |
Calcium (ionised) | 1.10 mmol/L | 1.03 – 1.23 |
Lactate | 4.1 mmol/L* | 0.6 – 2.4 |
Osmolality | 324 mOsm/kg* | 275 – 295 |
a) What is the most likely diagnosis? (10% marks)
b) What is the pathophysiology of the visual disturbance? (20% marks)
c) List three specific treatments you would institute. (15% marks)
a)
Methanol toxicity
b)
Methanol - > formaldehyde - > formate which is neurotoxic (especially retina and basal ganglia)
c)
Sodium bicarbonate
ADH inhibition with Ethanol (or fomepizole if available)
Dialysis
Cofactor therapy with either folic or folinic acid
So as to be fair to the other no-less-toxic alcohols, here is a table of the common alcohol toxidromes
Disorder | Toxin | Clinical and Laboratory Abnormalities |
---|---|---|
Alcoholic ketoacidosis |
|
|
Methanol intoxication |
|
|
Ethylene glycol intoxication |
|
|
Diethylene glycol intoxication |
|
|
Propylene glycol intoxication |
|
|
Isopropyl alcohol intoxication |
|
|
Management of toxic alcohol poisoning:
Decontamination
Enhanced elimination
Specific antidotes
Supportive management
Kraut, Jeffrey A., and Ira Kurtz. "Toxic alcohol ingestions: clinical features, diagnosis, and management." Clinical Journal of the American Society of Nephrology 3.1 (2008): 208-225.
Henderson, William R., and Jeffrey Brubacher. "Methanol and ethylene glycol poisoning: a case study and review of current literature." Cjem 4.1 (2002): 34-40.
a) Outline the clinical features and laboratory abnormalities likely to be found in a patient with envenomation due to an Australian snake-bite. (50% marks)
b) Outline the management of a patient with confirmed snake envenomation. (50% marks)
a)
Clinical features
Local pain, swelling and bruising. This may be absent
Sudden collapse – associated with hypotension and loss of consciousness, rarely cardiac arrest and seizure (5%)
Non –specific systemic symptoms – nausea, vomiting, diarrhoea, headache, sweating.
Neurotoxicity – descending flaccid paralysis – starting with ptosis, diplopia, blurred vision, and then progressing to bulbar weakness, respiratory and limb muscle paralysis.
Myotoxicity – local and generalised myalgia and muscle tenderness. Haemorrhage – rare – intracranial, gastrointestinal or from cannula sites
Laboratory abnormalities
Venom induced consumptive coagulopathy – characteristic of Australian snake bite – INR >3, APPT >100, fibrinogen < 1, raised D-dimers – can be 100 times assay cut off, Thrombocytopenia <100
CK – 1000 to over 100,000 u/L associated with myotoxicity
Acute renal failure – raised potassium, urea and creatinine.
Fragmented red cells in blood film – microangiopathic haemolytic anaemia.
b)
Management
First aid – Pressure bandage with immobilisation of the limb and the patient, pressure similar to that for a sprained ankle.
Monitor the patient in critical care area with resuscitation facilities – ED, HDU, ICU – neurological state, HR, BP, respiration, bleeding
Resuscitation as appropriate with two large bore cannulas and collect blood for laboratory tests – Coags (INR, APTT, Fibrinogen, D-Dimers), platelets, Urea, creatinine, electrolytes, CK.
Identify the likely snake type; the site of the bite can be swabbed and a venom detection kit (VDK) used or urine but not blood, or consultation with an herpetologist. Administer anti-snake venom (ASV) only if clinical symptoms or signs or lab abnormalities such prolonged INR. Current guidelines are for one vial ASV only and then correct subsequent coagulopathy with FFP
Release pressure bandage only after administration of ASV.
Type of ASV (monovalent or polyvalent) depends on clinical presentation, geography and VDK.
Monitor closely for anaphylactic reaction. Treat with adrenaline. Premedication with adrenaline, steroids or antihistamines not recommended.
Repeat lab investigations at 6, 12 and 24 hours to monitor response such as improvement in coagulopathy (INR).
Supportive treatment such ventilation for muscle paralysis and respiratory failure, dialysis for acute renal failure, inotropes for cardiovascular collapse and FFP for severe coagulopathy and bleeding complications
Specific clinical features
Laboratory findings and investigations for a snake bite victim:
Indications for polyvalent antidote:
Evidence for premedication for antivenom administration:
How do you know your monovalent antivenom is working?
Isbister, Geoffrey K., et al. "Snakebite in Australia: A practical approach to diagnosis and treatment." Medical journal of Australia 199.11 (2013): 763-768.
A 45-year-old male is admitted to the Emergency Department after ingesting an unknown quantity of“headache tablets”. His initial complaints are nausea, vomiting, shortness of breath and tinnitus. Fluid resuscitation has been commenced. You are asked to assess him as he is getting more dyspnoeic.
His serum biochemistry and arterial blood gas profile are as follows:
Parameter |
Patient Value |
Normal Adult Range |
|
Sodium |
138 mmol/L |
135 – 145 |
|
Potassium |
3.2 mmol/L* |
3.4 – 5.0 |
|
Chloride |
108 mmol/L |
100 – 110 |
|
Bicarbonate |
10 mmol/L* |
22 – 27 |
|
FiO2 |
0.3 |
||
pH |
7.32* |
7.35 – 7.45 |
|
PO2 |
125 mmHg (16.4 kPa) |
||
PCO2 |
20 mmHg (2.6 kPa)* |
35 – 45 (4.6 – 6.0) |
|
Base Excess |
-10 mmol/L* |
-2 – +2 |
|
Salicylate level |
105 mg/dL* |
3 – 10 |
|
Paracetamol level |
< 20 mg/L (< 130 µmol/L) |
< 20 (< 130) |
a) Describe the acid-base status. (20% marks)
b) What are four severe complications of this toxidrome? (20% marks)
c) What coagulopathy may be present in this toxidrome and what is the treatment? (10% marks)
d) What are the treatment options for severe toxicity, and what is their rationale? (50% marks)
a)
Acid-base status:
Increased anion gap metabolic acidosis Concomitant normal anion gap metabolic acidosis Respiratory alkalosis
Decreased delta ratio
b) Hypoglycaemia
Pulmonary oedema Cerebral oedema Arrhythmias Hyperpyrexia
c) Hypoprothrombinaemia Vitamin K
d)
Forced alkaline diuresis. Renal excretion of salicylates becomes important when the metabolic pathways become saturated. There is a 10-20 fold increase in elimination when the urine pH increased from 5 to 8.
Haemodialysis. Most of the drug is protein-bound, and is concentration dependant. The volume of distribution is small, and binding site saturation leads to large levels of free drug, which is easily dialyzable.
Multiple-dose charcoal. Many aspirin forms are slow release and after ingestion they clump together in the GI tract, forming a large slow release preparation. It is also poorly soluble in the stomach leading to delayed absorption.
Additional Examiners’ Comments:
Most candidates understood the acid-base abnormalities but not all were able to provide cogent answers relating to the complications and management. Few were able to describe all the treatment options for severe toxicity with the rationale for these strategies.
This question is identical to Question 10 from the second paper of 2012.
b) Complicatons of salicylate overdose:
Serum level 30-50mg/dL: | Serum level 50-75mg/dL: | Serum level >75mg/dL: |
|
|
|
b) Coagulopathy in salicylate overdose? Its not just platelet inhibition. According to UpToDate, this is because of hepatotoxicity and interference with the synthesis of vitamin K dependent factors. Specifically, it is well known that salicylate toxicity can cause a decrease in prothrombin. Vitamin K (if not prothrombinex) is the answer.
c)Management of sever salicylate overdose consists of the following measures:
Severe toxicity from salicylates has several treatment options:
Decontamination
Direct and indirect antidotes
Enhancement of clearance
Supportive ICU therapies
O'Malley, Gerald F. "Emergency department management of the salicylate-poisoned patient." Emergency medicine clinics of North America 25.2 (2007): 333-346.
Pinedo, H. M., L. B. van de Putte, and E. A. Loeliger. "Salicylate-induced consumption coagulopathy." Annals of the rheumatic diseases 32.1 (1973): 66.
Shapiro, Shepard, Milton H. Redish, and Harold A. Campbell. "Studies on Prothrombin: IV. The Prothrombinopenic Effect of Salicylate in Man."Experimental Biology and Medicine 53.2 (1943): 251-254.
Pearlman, Brian L., and Rashi Gambhir. "Salicylate Intoxication." Postgraduate medicine 121.4 (2009).
Outline the differences in the assessment and management of poisoning from substance ingestion in the following clinical scenarios, compared with a healthy young adult:
a) 2-year-old child. (30% marks)
b) 30-week gestation pregnant female. (35% marks)
c) 75-year-old adult with chronic kidney disease. (35% marks)
2-year-old child
30/40 pregnant female
75-year-old with CKD
This is another one of the questions in this paper which had a 0% pass rate. Locally available resources include the following chapters:
The answer would probably work better as a table:
Group | Infant/toddler | Pregnant woman | Elderly |
Absorption |
|
|
|
Distribution |
|
|
|
Clearance |
|
|
|
Pharmacodynamics |
|
|
|
Pattern of poisoning |
|
|
|
Differences in approach |
|
|
|
Kearns, Gregory L., et al. "Developmental pharmacology—drug disposition, action, and therapy in infants and children." New England Journal of Medicine 349.12 (2003): 1157-1167.
Barry, J. Dave. "Diagnosis and management of the poisoned child." Pediatric annals 34.12 (2005): 937-946.
Reid, David HS. "Treatment of the poisoned child." Archives of disease in childhood 45.241 (1970): 428.
Henretig, Fred M. "Special considerations in the poisoned pediatric patient." Emergency medicine clinics of North America 12.2 (1994): 549-567.
Calello, Diane P., and Fred M. Henretig. "Pediatric toxicology: specialized approach to the poisoned child." Emergency medicine clinics of North America 32.1 (2014): 29-52.
Shieh-Czaja, Angela, Diane P. Calello, and Kevin C. Osterhoudt. "Sick sisters." Pediatric emergency care 21.6 (2005): 400-402.
Anderson, Gail D. "Pregnancy-induced changes in pharmacokinetics." Clinical pharmacokinetics 44.10 (2005): 989-1008.
Goldfranks Manual of Toxicologic Emergencies: 2007 Edition, Ch. 30: "Reproductive and Perinatal Principles"
Zelner, Irene, et al. "Acute poisoning during pregnancy: observations from the toxicology investigators consortium." Journal of medical toxicology 11.3 (2015): 301-308.
Klein-Schwartz, Wendy, and Gary M. Oderda. "Poisoning in the elderly." Drugs & aging 1.1 (1991): 67-89.
Ticehurst, Stephen, et al. "Elderly patients with deliberate self-poisoning treated in an Australian general hospital." International psychogeriatrics 14.1 (2002): 97-105.
Carlsten, A., Margda Waern, and P. Allebeck. "Suicides by drug poisoning among the elderly in Sweden 1969–1996." Social psychiatry and psychiatric epidemiology 34.11 (1999): 609-614.
Jansen, Paul AF, and Jacobus RBJ Brouwers. "Clinical pharmacology in old persons." Scientifica 2012 (2012).
Sotaniemi, Eero A., et al. "Age and cytochrome P450-linked drug metabolism in humans: an analysis of 226 subjects with equal histopathologic conditions." Clinical pharmacology and therapeutics 61.3 (1997): 331-339.
Mitchell, Rebecca J., et al. "Dementia and intentional and unintentional poisoning in older people: a 10 year review of hospitalization records in New South Wales, Australia." International Psychogeriatrics 27.11 (2015): 1757-1768.
Rogers, Jody J., and Kennon Heard. "Does age matter? Comparing case fatality rates for selected poisonings reported to US poison centers." Clinical toxicology 45.6 (2007): 705-708.
Doak, Martin W., et al. "Self-poisoning in older adults: patterns of drug ingestion and clinical outcomes." Age and ageing 38.4 (2009): 407-411.
With respect to salicylate toxicity:
a) List four severe complications. (20% marks)
b) List the associated haematological abnormalities. (10% marks)
c) List the options for enhancing salicylate removal, and briefly explain the rationale for each option listed. (50% marks)
d} Give your interpretation of a declining serum salicylate level. (20% marks)
a) List four severe complications:
Additional Examiners‟ Comments:
Most candidates were able to give general statements but were unable to give specifics – in particular about how the therapies worked. There was poor understanding of the pharmacokinetics of salicylates and the rationale for the use of haemodialysis.
This question closely resembles Question 10 from the second paper of 2012 and the identical Question 17 from the second paper of 2015, except instead of asking about "what coaguloapthy might be present",
a)
Salicylate toxicity has a whole list of complications:
c)
c)
Severe toxicity from salicylates has several treatment options:
Decontamination
Direct and indirect antidotes
Enhancement of clearance
d) A declining salicylate level means nothing. Serial salicylate level measurement is meaningless, because:
Salicylate level may be declining because
O'Malley, Gerald F. "Emergency department management of the salicylate-poisoned patient." Emergency medicine clinics of North America 25.2 (2007): 333-346.
Pinedo, H. M., L. B. van de Putte, and E. A. Loeliger. "Salicylate-induced consumption coagulopathy." Annals of the rheumatic diseases 32.1 (1973): 66.
Shapiro, Shepard, Milton H. Redish, and Harold A. Campbell. "Studies on Prothrombin: IV. The Prothrombinopenic Effect of Salicylate in Man."Experimental Biology and Medicine 53.2 (1943): 251-254.
Pearlman, Brian L., and Rashi Gambhir. "Salicylate Intoxication." Postgraduate medicine 121.4 (2009).
Rothschild, Bruce M. "Hematologic perturbations associated with salicylate." Clinical Pharmacology & Therapeutics 26.2 (1979): 145-152.
Sanford-Driscoll, Marcia, and Leroy C. Knodel. "Induction of hemolytic anemia by nonsteroidal antiinflammatory drugs." Annals of Pharmacotherapy 20.12 (1986): 925-934.
Mandelli, M., and G. Tognoni. "Monitoring plasma concentrations of salicylate." Clinical pharmacokinetics 5.5 (1980): 424-440.
Done, Alan K. "SALICYLATE INTOXICATION Significance of Measurements of Salicylate in Blood in Cases of Acute Ingestion." Pediatrics 26.5 (1960): 800-807.
Kashani, John, and Richard D. Shih. "Salicylate Overdose." Encyclopedia of Intensive Care Medicine (2012): 2011-2014.
Please note: The following ECG has been recorded at 25 mm/sec and gain setting of 10 mm/mV.
A 73-year-old female collapsed in the Outpatient Radiology Department where she had been waiting to have a CT coronary angiogram. She had been given 160 mg verapamil to slow her heart rate for the scan.
Her usual medications included sotalol 80 mg twice a day.
On arrival of the Rapid Response Team she was drowsy, cold and peripherally shut down with systolic blood pressure 60 mmHg. Her arterial blood gas results at the scene are below, and her ECG is shown on page 3 (Figure 1).
Parameter |
Patient Value |
Adult Normal Range |
Fi02 |
0.5 |
|
pH |
7.05* |
7.35 — 7 45 |
pCO2 |
40.4 mmHg (5.3 kPa) |
35.0 - 45.0 (4.6 - 6.0) |
pO2 |
221 mmHg (29.1 kPa) |
|
SpO2 |
98% |
|
Bicarbonate |
10.5 mmol/L• |
22.0-26.0 |
Base Excess |
-17.9 mmol/l-• |
-2.0- +2 0 |
Lactate |
8.0 mmol/L* |
0.5- 1 6 |
Sodium |
132 rnmol/l_• |
135 - 145 |
Potassium |
5.4 mmol/L* |
3.5 - 5.0 |
Chloride |
105 mmol/L |
95 - 105 |
Glucose |
5.3 mmol/l- |
3.5 - 6.0 |
Give the likely underlying cause for the patient's collapse. (10% marks)
Interpret the investigations. (20% marks)
Outline specific therapies for the management of this patient, indicating the doses and mechanisms of action for any pharmacotherapy you have listed. (70% marks)
a) Cardio-toxicity from a combination of a beta-blocker and calcium channel blocker resulting in
cardiogenic shock.
Candidates may include a differential diagnosis – MI and cardiogenic shock not unreasonable.
b) Metabolic (lactic) acidosis with inadequate respiratory compensation
A-aDO2 approx 85 mmHg – raised for 73-year-old
Junctional bradycardia (but much slower than expected). Ventricular escape rhythm
acceptable. Peri arrest.
c) Specific therapies
Statement on resuscitation (Rapid ABC; iv access; O2, start CPR if indicated, monitor, rapid
echo).
Multiple agents often required with stepwise approach.
• Atropine 1mg stat (can be repeated x 3; often ineffective; muscarinic receptor
antagonist increases SA node discharge, conduction through the AV node and opposes
action of Vagus nerve)
• Adrenaline or Noradrenaline infusion starting at 10-20 g/min and titrate to a MAP > 65
mmHg (+ve inotropy, chronotropy, vasoconstriction)
• Calcium – Chloride or Gluconate can be given (more calcium in CaCl) – 10mls of 10%
solution (can be repeated x3 +/- infusion; competitively increases calcium entry into the
myocardium via non-blocked channels)
• Glucagon 5mg stat (can be repeated x3; increases intracellular cAMP and has been
shown to increase heart rate in BOTH beta-blocker and CCB toxicity).
• 100mls 8.4% NaHCO3 stat (she is already very acidotic)
• Hyperinsulinaemia-Euglycaemia – short acting insulin 1 unit/kg with 50mls 50%
Dextrose bolus, then 0.5 units insulin /kg/hr with 10% dextrose infusion and q1hrly BGLs
and K+ (high dose insulin = +ve inotrope but mechanism not clearly understood)
• Lipid Emulsion – 1ml/kg 20% lipid emulsion bolus (can be repeated x 3 then start
infusion 0.5mls/kg/min; acts as a “lipid sink” surrounding lipophillic drugs rendering them
ineffective & maybe fatty energy source for myocardium)
Other Therapies
• Trans-cutaneous pacing
• Trans-venous temporary pacing.
• VA-ECMO
Additional Examiners’ Comments:
Many candidates failed to interpret the ECG, or to discuss the mechanism of therapies. Basic
knowledge gaps in many answers.
The image used in this SAQ is not from the original college paper (those are a sacred and jealously guarded resource). Fortunately, the ECG I found at LITFL is virtually identical.
In short, the underlying cause of the collapse is simultaneous calcium channel blocker and beta blocker overdose, a variant on the theme of toxic antiarrhythmic polypharmacy.
To interpret the investigations:
As to management: the college asked to outline specific therapies, i.e. not supportive ones. For the management of combined beta blocker and calcium channel blocker toxicity, this would really consist of the following strategies:
Management | Dose and rationale |
Decontamination with activated charcoal |
1g/kg of charcoal; because it may slow the absorption of some beta-blockers and calcium channel blockers. |
Calcium infusion |
0.2mmol/hr. |
High-dose insulin |
0.5-2.0 unit/kg/hr, as well as whatever amount of dextrose is required to maintain normoglycaemia. This is becoming the standard of care (Woodward et al, 2014) Why?
|
Glucagon |
May be effective (eg. Doyon et al, 1993) but is not recommended as a first-line agent. |
Intralipid |
Lipid emulsion should "decontaminate" the bloodstream by making these highly lipophilic drugs less bioavailable (true for most of them, with the exception of atenolol and sotalol). Verapamil toxicity is listed as one of the indications for the use of lipid emulsion in toxicology(Cave and Harvey, 2009). |
In addition, generic supportive therapies can be listed, although they may not attract any marks:
Nawrath, H., et al. "Class I Antiarrhythmic Drug Effects: What Is the Basis for Subgroups Ia, Ib and Ic." Cardiac Arrhythmias: The Management of Atrial Fibrillation (2013): 39.
Vaughan-Williams, E. M. "Classification of antiarrhythmic drugs." Cardiac arrhythmias 449 (1970).
Rosen, Michael R. "The sicilian gambit-a new approach to the classification of antiarrhythmic drugs based on their actions on arrhythmogenic mechanisms." Circulation 84.4 (1991): 1831-1851.
Kowey, Peter R. "Pharmacological effects of antiarrhythmic drugs: Review and update." Archives of internal medicine 158.4 (1998): 325-332.
Siddoway, L. A., P. J. Podrid, and P. R. Kowey. "Pharmacologic principles of antiarrythmic drugs." (1995): 355-368.
Yamreudeewong, Weeranuj, et al. "Potentially significant drug interactions of class III antiarrhythmic drugs." Drug safety 26.6 (2003): 421-438.
Strauss, William E., and Alfred F. Parisi. "Combined Use of Calcium-Channel and Beta-Adrenergic Blockers for the Treatment of Chronic Stable AnginaRationale, Efficacy, and Adverse Effects." Annals of internal medicine 109.7 (1988): 570-581.
Markota, Andrej, et al. "Treatment of near-fatal beta blocker and calcium channel blocker intoxication with hyperinsulinemic euglycemia, intravenous lipid emulsions and high doses of norepinephrine." Signa Vitae 10.1 (2015): 144-150.
Woodward, Christina, Ali Pourmand, and Maryann Mazer-Amirshahi. "High dose insulin therapy, an evidence based approach to beta blocker/calcium channel blocker toxicity." Daru 22.36 (2014): 2008-223.
Henry, Philip D. "Comparative pharmacology of calcium antagonists: nifedipine, verapamil and diltiazem." The American journal of cardiology 46.6 (1980): 1047-1058.
Doyon, Suzanne, and James R. Roberts. "The use of glucagon in a case of calcium channel blocker overdose." Annals of emergency medicine 22.7 (1993): 1229-1233.
A 65-year old male has been admitted to ICU needing invasive mechanical ventilation following two episodes of generalised tonic-clonic convulsions and vomiting after an episode of suspected self-harm.
He has a history of hypertension, chronic obstructive pulmonary disease (COPD) and depression. His medications include Ramipril,Fluoxetine,Metoprolol,Theophylllne and Fluticasone/Salmeterol inhaler.
His vital parameters are as follows:
His CT brain scan did not reveal any abnormality. Results of his biochemistry are as follows:
Parameter |
Patient Value |
Adult Normal Range |
|
Sodium |
136 mmol/l |
135 - 145 |
|
Potassium |
2.9 mmoVL* |
3.5 .5.5 |
|
Chloride |
105 mmol/L |
92 - 107 |
|
Bicarbonate |
10.9 mmoUL• |
22.0 - 28.0 |
|
Urea |
19.7 mmoll• |
2.5 - 6.5 |
|
Creatlnine |
220 umolJL• |
45 - 90 |
|
Magnesium |
0.55 mmovL· |
0 65 - 1.00 |
|
Phosphate |
0.55 mrnol/L* |
0.75 - 1.50 |
|
Corrected Calcium |
2.67 mrnol/l* |
2.15 -2.55 |
|
Creatinine Kinase |
150 U/L |
55 - 170 |
|
Blood Glucose |
15.2 mmol/l* |
3.5 - 6.0 |
|
Lactate |
4.9 mmol/L* |
< 2.0 |
Give the most likely diagnosis AND your reasoning. (40% marks)
Briefly outline your therapeutic strategies for this patient.
a)
• Acute Theophylline Poisoning. The clinical findings of vomiting, seizures, hypotension,
Atrial Flutter combined with metabolic abnormalities strongly suggests theophylline
poisoning
• Above biochemical abnormalities may suggest β-agonist toxicity; but cardiac arrhythmias
and seizures are rare features of β-agonist toxicity
• Biochemical findings and ECG abnormalities do not favour tricyclic anti-depressant or
SSRI overdose
b)
• Check serum theophylline
• Repeated doses of activate charcoal, as means of decontamination. Theophylline is also
more rapidly cleared from the blood in patients receiving activated charcoal
• Extracorporeal removal such as charcoal hemoperfusion or hemodialysis, as
theophylline has low volume of distribution without extensive protein binding. High
efficiency hemodialysis as effective as charcoal hemoperfusion
• Control of seizures with benzodiazepines. Phenytoin should be avoided as it is not
effective and may worsen mortality
• Correction of electrolyte abnormalities (hypokalemia, hypomagnesemia and
hypophosphatemia)
• IV Esmolol or amiodarone for cardiac arrhythmia, after correction of electrolyte
abnormalities
• Hypotension should be treated with IV fluids and/or noradrenaline. IV propranolol or
esmolol may reverse hypotension as it is caused by β2-adrenergic effects
• Hypercalcemia usually responds to fluid resuscitation
• Hyperglycemia responds to fluids and/or insulin administration
Additional Examiner Comments:
Several candidates failed to recognise theophylline poisoning. Many candidates failed to read the stem and did not give a rationale for their diagnosis. Management of theophylline toxicity was discussed poorly.
Let us interpret these data systematically.
So, sounds like a theophylline overdose. As the collegely rightly pointed out, there is no way this old guy could have cosumed enough salmeterol to make him this sick.
In general, the features of theophylline overdose are as follows:
Symptoms | Signs | Biochemistry |
|
|
|
As for the management:
Decontamination
Enhanced elimination
Antidotes
Supportive management
A - the patient will likely need intubation at some stage
B - ventilate them with a slightly higher rate to maintain the compensation for metabolic acidosis
C - they will likely be hypotensive with a large overdose; noradrenaline will be required.
They will also have arrhythmias. The college answer helpfully suggests esmolol or amiodarone. Esmolol has been used successfully (Seneff et al, 1990) and may paradoxically improve blood pressure by acting as a β2-antagonist, as well as slowing the rate and improving diastolic filling.
D - Sedation with benzodiazepines seems like a sensible move.
Likely, the patient will need them anyway for seziure control.
Other antiepileptics are apparently ineffective.
E - Correct all their electrolyte disturbances
F - Consider dialysis; high efficiency dialysis may even remove some theophylline
G - Regular antiemetics and/or NGT (given how much you are relying on multi-dose charcoal)
Barnes, Peter J. "Theophylline." American journal of respiratory and critical care medicine 188.8 (2013): 901-906.
Hendeles, Leslie, et al. "Food-induced “dose-dumping” from a once-a-day theophylline product as a cause of theophylline toxicity." Chest 87.6 (1985): 758-765.
Ehlers, Sally M., Darwin E. Zaske, and Ronald J. Sawchuk. "Massive theophylline overdose: Rapid elimination by charcoal hemoperfusion." Jama240.5 (1978): 474-475.
Hall, Kevin W., et al. "Metabolic abnormalities associated with intentional theophylline overdose." Annals of internal medicine 101.4 (1984): 457-462.
Seneff, Michael, et al. "Acute theophylline toxicity and the use of esmolol to reverse cardiovascular instability." Annals of emergency medicine 19.6 (1990): 671-673.
MILTON, L. McPHERSON, et al. "Theophylline-lnduced Hypercalcemia."Annals of internal medicine 105 (1986): 52-54.
You have received a phone call from a junior colleague at a remote location. A previously well 32-year-old male has presented with nausea and hypotension following a confirmed bite on his leg from a brown snake. A retrieval team will arrive in approximately three hours; until then your colleague is the only medical officer available.
a) Outline the telephone advice you would give them. Include guidance on what complications they might expect to arise and how to manage them. (80% marks)
b) Several days after arrival in your Intensive Care Unit (ICU) the patient develops oliguric renal failure. List the possible causes. (20% marks)
a)
Complications include:
b)
Examiners Comments:
Many candidates ignored the setting of a remote location completely, and gave a management plan that was applicable to a tertiary centre (e.g., TEG and ROTEM; "intubate" without reference to the skill of the junior doctor, etc.).
Some candidates appeared unaware of even the most basic aspects of snake bite management e.g., pressure immobilization, VDK, monovalent versus polyvalent etc.
Many candidates used an ABCDE template which prioritized airway and breathing above the first-aid of snake bite; also, it resulted in not covering the coagulopathy aspects well enough.
The answer for the renal failure again seemed templated (pre-renal, renal, post-renal) and lacked context - there were very few references to the snake bite and antivenom as possible causes of renal failure
The venom itself is a mixture of presynaptic and postsynaptic neurotoxins and procoagulants. There is nothing myotoxic or nephrotoxic in the venom. Acute kidney injury is seen anyway because of thrombotic microangiopathy, which is a side-effect of the procoagulant venom.
Brown snake venom produces the following stereotypical effects:
Specific management steps should include:
Distant back-of-Bourke management should consist of:
ICU-level management should consist of the following supportive steps:
Though the examiners complained bitterly about templated answers being used to mask the candidates' unfamiliarity with snake bites, one cannot help but note that in the absence of specific venom nephrotoxins the patient's renal failure could be due to any of the normal things which cause renal failure. And these things are typically categorised as pre-renal, post-renal and intra-renal. With the exception of VICC-induced microangiopathy, the college list of differentials is certainly no different to a normal list of causes for renal failure in critical illness, featuring such favourites as "sepsis" and "ATN secondary to prolonged hypotension/arrest". In response, here is a classically organised list of plausible-sounding reasons for renal failure in a patient with a brown snake bite:
Pre-renal |
Intra-renal |
Post-renal |
|
|
|
Isbister, Geoffrey K., et al. "Snakebite in Australia: a practical approach to diagnosis and treatment." Med J Aust 199 (2013): 763-768.
Bücherl, Wolfgang, Eleanor E. Buckley, and Venancio Deulofeu, eds. Venomous Animals and Their Venoms: Venomous Vertebrates. Vol. 1. Elsevier, 2013.
Russell, Findlay E., and Harold W. Puffer. "Pharmacology of snake venoms." Clinical toxicology 3.3 (1970): 433-444.
Daltry, Jennifer C., Wolfgang Wüster, and Roger S. Thorpe. "Diet and snake venom evolution." Nature 379.6565 (1996): 537-540.
Allen, George E., et al. "Clinical effects and antivenom dosing in brown snake (Pseudonaja spp.) envenoming—Australian snakebite project (ASP-14)." PLoS One 7.12 (2012): e53188.
Brown, Simon GA, et al. "Clotting factor replacement and recovery from snake venom-induced consumptive coagulopathy." Intensive care medicine 35.9 (2009): 1532-1538.
Isbister, Geoffrey K., et al. "Thrombotic microangiopathy from Australian brown snake (Pseudonaja) envenoming." Internal medicine journal 37.8 (2007): 523-528.
Tibballs, J., et al. "The cardiovascular and haematological effects of purified prothrombin activator from the common brown snake (Pseudonaja textilis) and their antagonism with heparin." Anaesthesia and intensive care 20.1 (1992): 28-32.
Ponraj, Durairaj, and Ponnambalam Gopalakrishnakone. "Establishment of an animal model for myoglobinuria by use of a myotoxin from Pseudechis australis (king brown snake) venom in mice." Laboratory animal science 46.4 (1996): 393-398.
White, Julian. "Factor replacement for Australian snakebite coagulopathy: a re-evaluation?." (2009): Intensive Care Med (2009) 35:1503–1504
In the setting of haemodynamic collapse secondary to drug overdose, give the pharmacological antidote/s for each of the agents listed below. For each antidote cited, give the rationale/mechanism of action.
a) Digoxin.
b) Tricyclic anti-depressants.
c) Beta blockers.
d) Lignocaine.
Detail in template more than required for full marks:
Digoxin |
Digoxin Fab Fragments (Digibind) |
|
TCA |
Sodium bicarbonate |
Sodium load
|
Beta Blockers |
Glucagon |
|
High Dose insulin +/- glucose therapy |
Several theories of effect:
|
|
Atropine |
• Anti-cholinergic agent |
|
Lignocaine |
Lipid emulsion therapy |
|
This question begs for a tabulated answer. The college table is comprehensive and difficult to improve upon. One's only recourse would be either to make the answer more succinct, or (more likely) to add more unnecessary detail ("more than required for full marks").
Drug | Antidote | Rationale/mechanism |
Digoxin |
Digoxin-specific Fab fragments |
|
Tricyclics | Sodium bicarbonate |
|
β-blockers | High dose insulin with euglycaemia |
|
Glucagon |
|
|
Atropine |
|
|
Lignocaine | Lipid emulsion |
|
Hauptman, Paul J., and Ralph A. Kelly. "Digitalis." Circulation 99.9 (1999): 1265-1270.
Hoffman, J. R., and C. R. McElroy. "Bicarbonate therapy for dysrhythmia and hypotension in tricyclic antidepressant overdose." Western Journal of Medicine134.1 (1981): 60.
Woodward, Christina, Ali Pourmand, and Maryann Mazer-Amirshahi. "High dose insulin therapy, an evidence based approach to beta blocker/calcium channel blocker toxicity." Daru 22.36 (2014): 2008-223.
Donald, M. J., and S. Derbyshire. "Lignocaine toxicity; a complication of local anaesthesia administered in the community." Emergency medicine journal 21.2 (2004): 249-250.
Chan, B. S. H., and N. A. Buckley. "Digoxin-specific antibody fragments in the treatment of digoxin toxicity." Clinical Toxicology 52.8 (2014): 824-836.
Shepherd, Greene. "Treatment of poisoning caused by β-adrenergic and calcium-channel blockers." American Journal of Health-System Pharmacy 63.19 (2006): 1828-1835.
Engebretsen, Kristin M., et al. "High-dose insulin therapy in beta-blocker and calcium channel-blocker poisoning." Clinical toxicology (2011).
Peterson, Charles D., J. Steven Leeder, and Steve Sterner. "Glucagon therapy for β-blocker overdose." Drug intelligence & clinical pharmacy 18.5 (1984): 394-398.
Ciechanowicz, Sarah, and Vinod Patil. "Lipid emulsion for local anesthetic systemic toxicity." Anesthesiology research and practice 2012 (2012).
Weinberg, Guy L., et al. "Pretreatment or resuscitation with a lipid infusion shifts the dose-response to bupivacaine-induced asystole in rats." The Journal of the American Society of Anesthesiologists 88.4 (1998): 1071-1075.
Discuss the pathophysiology, clinical features and the management of a patient who presents with acute crystal methamphetamine ("ICE") intoxication
Pathophysiology
Clinical features
Candidates should have demonstrated an understanding of the multisystem nature of the condition (e.g. listing of several affected systems) in order to score well for this section.
Management
Examiners Comments:
A number of candidates only mentioned generic details in their answer instead of specific issues related to the condition. Knowledge of the pathophysiology was poor.
Pathophysiology:
Clinical features:
Management:
Li, Wenlong, and Naren Gunja. "Illicit drug overdose: Prevalence and acute management." Australian family physician 42.7 (2013): 481.
Vasan, Sarayu, and Garth J. Olango. "Toxicity, Amphetamine." (2017).
Richards, John, and Erik Laurin. "Toxicity, methamphetamine." (2017).
Darke, Shane, Sharlene Kaye, and Johan Duflou. "Rates, characteristics and circumstances of methamphetamine‐related death in Australia: a national 7‐year study." Addiction112.12 (2017): 2191-2201.
Albertson, Timothy E., Robert W. Derlet, and Brent E. Van Hoozen. "Methamphetamine and the expanding complications of amphetamines." Western Journal of Medicine 170.4 (1999): 214.
King, Andrew, Mirjana Dimovska, and Luke Bisoski. "Sympathomimetic Toxidromes and Other Pharmacological Causes of Acute Hypertension." Current hypertension reports20.1 (2018): 8.
Laitselart, Philippe, et al. "Severe Sympathomimetic Toxidrome in a French Soldier: How Caffeine Overdose Can Lead to Severe Consequences." Military Medicine (2017).
Richards, John R., et al. "Treatment of toxicity from amphetamines, related derivatives, and analogues: a systematic clinical review." Drug & Alcohol Dependence 150 (2015): 1-13.
Jenner, L., et al. "Management of patients with psychostimulant toxicity: guidelines for emergency departments." Canberra, Australian Government Department of Health and Ageing (2006).
Compare and contrast Serotonin Syndrome with Neuroleptic Malignant Syndrome
Serotonin syndrome (SS) |
Neuroleptic malignant syndrome (NMS) |
|
Precipitants & Risk factors |
Serotonergic Agents such as TCAs, SSRIs, SNRIs, MAOIs, triptans, nefazodone, buspirone, mirtazapine, carbamazepine, tramadol, linezolid, MDMA (ecstasy), dextromethorphan, St. John's wort, lithium, methadone, cocaine, levodopa, reserpine, and amphetamines. *naming a few drugs/classes adequate Usually concurrent use of multiple agents |
Dopamine Antagonists such as antipsychotics and antiemetics. Also, abrupt withdrawal of dopamine agonists, for instance, those used in the management of Parkinson's disease, may produce signs and symptoms correlating with NMS. NMS does not necessarily correspond with high doses of antipsychotics, as it can occur with lower doses |
Concurrent use of serotonergic agents Use of illicit drugs, especially when used in patients concurrently taking a serotonin enhancing drug. |
Use of first- &/or second-generation antipsychotics. Use of higher doses of first- &/or second-generation antipsychotics Rapid escalation of dosing, switching among agents, higher potency agents, and long-acting depot formulations |
|
Incidence |
Rare |
0.02–2.4% in patients being treated with neuroleptics |
Time of onset following inciting agent |
Usually < 24 hours of initiation or change in a medication |
Usually 1-3 days (can be later) of exposure to a dopamine antagonist or withdrawal of a dopamine agonist |
Autonomic features |
Tachypnoea Hyperthermia (> 40°C) Tachycardia Hypertension Diaphoresis Hypersalivation |
Tachypnoea Hyperthermia (> 40°C) Tachycardia Hypertension Diaphoresis Hypersalivation |
Neuromuscular |
Increased tone, worse in the lower extremities than upper extremities Hyperreflexia Clonus (unless masked by increased muscle tone) Dilated pupils Classically agitation then coma |
'Lead-pipe' rigidity globally Rapid, increasing signs of extrapyramidal symptoms Hyporeflexia Normal pupils Classically alert then coma |
Treatment |
Discontinue serotonergic agents Benzodiazepines Cyproheptadine Supportive management |
Discontinue dopaminergic agents Cooling Fluids Benzodiazipines Dopamine agonists e.g. Bromocriptine or amantidine Dantrolene Supportive management |
Examiners Comments:
Marks were allocated to descriptions of Precipitants and Risk factors, Clinical Features/Diagnosis and Management – the specific headings in the Table were not required.
Many candidates lacked the basic knowledge to pass the question, and many did not complete it. Many confused Neuroleptic Malignant Syndrome with Malignant Hyperthermia.
As a "compare and contrast" question, this one would benefit from a tabulated answer. The college table is of a sufficiently high quality that any attempt to "improve" on it would only lead to a messier more confusing answer model. As such, it would be completely consistent with the spirit of this revision resource.
SS | NMS | |
Causative agents | Serotonin agonists or antagonists | Dopamine antagonists or withdrawal of dopamine agonists |
Onset | Rapid (hours) | Gradual (days) |
Relationship to drug dose | Usually overdose or the effect of using a combination of several agents | Can occur with normal dosing, even after years of treatment with the same agent |
Level of consciousness | Agitation, hypervigilance, delirium | Encephalopathy, stupour, coma, mutism |
Pupils | Dilated | Normal |
Other cranial nerves | Usually unaffected | Dysphagia, aspiration |
Tone | Increased | Increased ("lead pipe") |
Reflexes | Increased | Decreased |
Clonus | Present (a diagnostic discriminator) | Absent |
Temperature | Raised | Raised |
Mucosa | Siallorhoea | Siallorhoea |
Cardiovascular findings | Tachycardia and hypertension | Haemodynamically unstable, may be either high or low |
Biochemistry | Rhabdomyolysis; CK rise | Rhabdomyolysis, CK rise Low serum iron |
Acid-base | Normal | Acidosis |
Haematology | May be normal | Raised white cell count |
Bowel sounds | Vigorously hyperactive | Reduced, sluggish |
Management | Cyproheptadine, olanzapine, chlorpromazine | Amantadine, bromocryptine, dantrolene |
Kateon, Hayley. "Differentiating serotonin syndrome and neuroleptic malignant syndrome." Mental Health Clinician 3.3 (2013): 129-133.
Nimmagadda, Seshagiri Rao, David Hugh Ryan, and Stephen Lawrence Atkin. "Neuroleptic malignant syndrome after venlafaxine." The Lancet 355.9200 (2000): 289-290.
Dunkley, E. J. C., et al. "The Hunter Serotonin Toxicity Criteria: simple and accurate diagnostic decision rules for serotonin toxicity." Qjm 96.9 (2003): 635-642.
Sternbach, Harvey. "The serotonin syndrome." The American journal of psychiatry 148.6 (1991): 705.
Lappin, Richard I., and Elizabeth L. Auchincloss. "Treatment of the serotonin syndrome with cyproheptadine." New England Journal of Medicine 331.15 (1994): 1021-1022.
Graudins, Andis, Andrew Stearman, and Betty Chan. "Treatment of the serotonin syndrome with cyproheptadine." Journal of Emergency Medicine 16.4 (1998): 615-619.
Gillman, P. K. "The serotonin syndrome and its treatment." Journal of Psychopharmacology 13.1 (1999): 100-109.
Jensen, Klaus. "The effect of antiserotonin (cyproheptadine) and antihistamine on cutaneous allergy." Allergy 15.4 (1960): 293-305.
Davis, John M., et al. "Electroconvulsive therapy in the treatment of the neuroleptic malignant syndrome." Convulsive therapy (1991).
Granato, Jerome E., et al. "Neuroleptic malignant syndrome: successful treatment with dantrolene and bromocriptine." Annals of neurology 14.1 (1983): 89-90.
A 25-year-old female is admitted with a reduced level of consciousness and suffers a brief seizure in the Emergency Department.
Her ECG is shown on page 14 (ECG 15.2).
a) Describe the abnormalities. (20% marks)
b) What is the most likely diagnosis? What urgent treatment is required and what is the mechanism of action of the treatment? (30% marks)
c) Which drug would you avoid using to treat her convulsion and why? (10% marks)
ECG 15.2
a)
Broad QRS complex, first degree heart block, prolonged QT, dominant R wave in AVR
b)
Sodium bicarbonate is used to treat a suspected TCA overdose. Alkalinization increases the binding of TCA to plasma proteins reducing the amount of free drug and reduces the amount of ionisation of the drug reducing its ability to pass through cell membranes. Also reduces extracellular K concentration, causing hyperpolarisation and reducing the Na channel blockage.
c)
Phenytoin (Class 1b) should be avoided that it would potentiate sodium channel blockade
Though it is impossible to guess which specific image the college used, one might be able to recapture their steps by googling "TCA overdose ECG" and picking out a top image result. That would probably end up being this classic ECG from LIFTL which was stolen and shamelessly re-posted here. The features of a TCA overdose listed there include:
Question 28.2 from the second paper of 2009 also asked specifically about "mechanism of effectiveness of sodium bicarbonate in the management of tricyclic antidepressant overdose." In summary, bicarbonate in TCA overdose works in the following ways:
The college focused on phenytoin in this scenario because the patient had a brief seizure, which brings up the question of antiepileptic choice. Phenytoin is such a good sodium channel blocker that it could be considered a Class I antiarrhythmic agent and a valid third line drug for refractory VT storm. What is more peculiar, is that of the currently available antiepileptic drugs, most either have a distince sodium channel blocker effect, or are thought to influence sodium channels in some other ambiguous round-about way. For instance, a 2017 paper by Brodie lists "phenytoin, carbamazepine, lamotrigine, oxcarbazepine, rufinamide, lacosamide and eslicarbazepine acetate" as classical sodium channel blockers. Sodium valproate, topiramate and zonisamide are also thought to have some sort of stabilising effect on sodium channels.
Even more interesting is the fact that that historically phenytoin was proposed as the treatment to reverse cardiotoxicity due to TCA overdose. Hagerman & Hanashiro (1981) confessed to administering 50mg/min to five adults, to a total dose of about 5-7mg/kg. All conduction defects disappeared within about three quarters of an hour. So, in the 1980s, phenytoin was felt to inhibit the sodium channel blockade effects, rather than potentiating them. Even still, in 2010, Foianini et al recommended the use of Class I agents (lignocaine more so than phenytoin) in severe TCA cardiotoxicity. It appears that these drugs are indicated in cases where the cardiotoxicity is refractory to treatment with sodium bicarbonate or hypertonic saline, or in which these sodium-rich substances are contraindicated (metabolic alkalosis or hypernatremia).
Even more interestingly,
Hoffman, J. R., and C. R. McElroy. "Bicarbonate therapy for dysrhythmia and hypotension in tricyclic antidepressant overdose." Western Journal of Medicine134.1 (1981): 60.
Kerr, G. W., A. C. McGuffie, and S. Wilkie. "Tricyclic antidepressant overdose: a review." Emergency Medicine Journal 18.4 (2001): 236-241.
Brown, T. C., et al. "The use of sodium bicarbonate in the treatment of tricyclic antidepressant-induced arrhythmias." Anaesthesia and intensive care 1.3 (1973): 203-210.
McCabe, James L., et al. "Experimental tricyclic antidepressant toxicity: a randomized, controlled comparison of hypertonic saline solution, sodium bicarbonate, and hyperventilation." Annals of emergency medicine 32.3 (1998): 329-333.
Bou-Abboud, Elias, and Stanley Nattel. "Molecular mechanisms of the reversal of imipramine-induced sodium channel blockade by alkalinization in human cardiac myocytes." Cardiovascular research 38.2 (1998): 395-404.
Hoffman, Jerome R., et al. "Effect of hypertonic sodium bicarbonate in the treatment of moderate-to-severe cyclic antidepressant overdose." The American journal of emergency medicine 11.4 (1993): 336-341.
Dargan, Paul I., Mark G. Colbridge, and Alison L. Jones. "The management of tricyclic antidepressant poisoning." Toxicological reviews 24.3 (2005): 187-194.
Kingston, Michael E. "Hyperventilation in tricyclic antidepressant poisoning." Critical care medicine 7.12 (1979): 550-551.
Wrenn, Keith, Brian A. Smith, and Corey M. Slovis. "Profound alkalemia during treatment of tricyclic antidepressant overdose: a potential hazard of combined hyperventilation and intravenous bicarbonate." The American journal of emergency medicine 10.6 (1992): 553-555.
Brodie, Martin J. "Sodium channel blockers in the treatment of epilepsy." CNS drugs 31.7 (2017): 527-534.
Hagerman, Gordon A., and Paul K. Hanashiro. "Reversal of tricyclic-antidepressant-induced cardiac conduction abnormalities by phenytoin." Annals of emergency medicine10.2 (1981): 82-86.
Foianini, Anthony, Timothy Joseph Wiegand, and Neal Benowitz. "What is the role of lidocaine or phenytoin in tricyclic antidepressant-induced cardiotoxicity?." Clinical Toxicology48.4 (2010): 325-330.
What are the biochemical findings in methanol toxicity? Outline the specific management along with its physiological rationale. (50% marks)
High anion gap metabolic acidosis, osmolar gap, elevated plasma methanol level.
Antidote therapy, often using ethanol or fomepizole, is directed towards delaying methanol metabolism until the methanol is eliminated from the patient’s system either naturally or via dialysis. Like methanol, ethanol is metabolized by ADH, but the enzyme’s affinity for ethanol is 10-20 times higher than it is for methanol. Fomepizole is also metabolized by ADH; however, its use is limited because of high cost and lack of availability
Dialysis: The toxic products of methanol and ethanol are formic acid and oxalic acid respectively. They are small molecules, are not protein bound and have low volume of distribution so are easily dialysable.
Folic acid – can accelerate the metabolism of formate via tetrahydrofolate.
The characteristic features of toxic alcohol toxicity in general are:
As for specific management:
Decontamination
Enhanced elimination
Specific antidotes
Morrow, Gregory P., et al. "In vivo kinetics of formate metabolism in folate-deficient and folate-replete rats." Journal of Biological Chemistry 290.4 (2015): 2244-2250.
Kraut, Jeffrey A., and Ira Kurtz. "Toxic alcohol ingestions: clinical features, diagnosis, and management." Clinical Journal of the American Society of Nephrology 3.1 (2008): 208-225.
Henderson, William R., and Jeffrey Brubacher. "Methanol and ethylene glycol poisoning: a case study and review of current literature." Cjem 4.1 (2002): 34-40.
Hovda, Knut Erik, Petter Urdal, and Dag Jacobsen. "Increased serum formate in the diagnosis of methanol poisoning." Journal of analytical toxicology 29.6 (2005): 586-588.
With respect to salicylate toxicity:
a) List four severe complications. (20% marks)
b) List the associated haematological abnormalities. (10% marks)
c) List the options for enhancing salicylate removal and briefly explain the rationale for each option listed. (50% marks)
d) When assessing a patient with salicylate toxicity, how would you interpret a declining serum salicylate level? (20% marks)
a)
List four severe complications
Pulmonary oedema
Cerebral oedema
Arrhythmias
Hyperpyrexia
Shock and cardiovascular collapse
Acid-base disturbance (high anion gap metabolic acidosis and respiratory alkalosis)
b)
List the associated haematological abnormalities Hypoprothrombinaemia
Thrombocytopaenia
c)
List the options for enhancing salicylate removal, and briefly outline the rational for each option listed:
Haemodialysis. Most of the drug is protein-bound, and is concentration dependant. The volume of distribution is small, and binding site saturation leads to large levels of free drug, which is easily dialyzable
Multiple-dose charcoal. Many aspirin forms are slow release and after ingestion they clump together in the GI tract, forming a large slow release preparation. It is also poorly soluble in the stomach leading to delayed absorption.
Forced alkaline diuresis. Renal excretion of salicylates becomes important when the metabolic pathways become saturated. There is a 10-20x increase in elimination when the urine pH increased from 5 to 8. Current role is questionable as haemodialysis is more efficient at removal, with less metabolic disturbance. Reasonable as initial therapy whilst waiting for circuit prime and line insertion.
d)
Give your interpretation of a declining serum salicylate level
It may indicate that the drug is moving into the tissues, and not necessarily being eliminated This means that clinical assessment is paramount
This question is identical to Question 8 from the second paper of 2016, except that the wording of section (d) is slightly different for some reason.
a)
Salicylate toxicity has a whole list of complications:
c)
c)
Severe toxicity from salicylates has several treatment options:
Decontamination
Direct and indirect antidotes
Enhancement of clearance
d) A declining salicylate level means nothing. Serial salicylate level measurement is meaningless, because:
Salicylate levels may be declining because
O'Malley, Gerald F. "Emergency department management of the salicylate-poisoned patient." Emergency medicine clinics of North America 25.2 (2007): 333-346.
Pinedo, H. M., L. B. van de Putte, and E. A. Loeliger. "Salicylate-induced consumption coagulopathy." Annals of the rheumatic diseases 32.1 (1973): 66.
Shapiro, Shepard, Milton H. Redish, and Harold A. Campbell. "Studies on Prothrombin: IV. The Prothrombinopenic Effect of Salicylate in Man."Experimental Biology and Medicine 53.2 (1943): 251-254.
Pearlman, Brian L., and Rashi Gambhir. "Salicylate Intoxication." Postgraduate medicine 121.4 (2009).
Rothschild, Bruce M. "Hematologic perturbations associated with salicylate." Clinical Pharmacology & Therapeutics 26.2 (1979): 145-152.
Sanford-Driscoll, Marcia, and Leroy C. Knodel. "Induction of hemolytic anemia by nonsteroidal antiinflammatory drugs." Annals of Pharmacotherapy 20.12 (1986): 925-934.
Mandelli, M., and G. Tognoni. "Monitoring plasma concentrations of salicylate." Clinical pharmacokinetics 5.5 (1980): 424-440.
Done, Alan K. "SALICYLATE INTOXICATION Significance of Measurements of Salicylate in Blood in Cases of Acute Ingestion." Pediatrics 26.5 (1960): 800-807.
Kashani, John, and Richard D. Shih. "Salicylate Overdose." Encyclopedia of Intensive Care Medicine (2012): 2011-2014.
A normally well 19-year-old female (65 kg) is admitted to your ICU after she had an intentional ingestion of 50 tablets of (her mother's) verapamil 180 mg (sustained release). The ingestion was 4 hours ago.
On admission, she is conscious, feels lightheaded, and has a heart rate of 40 beats/minute and a blood pressure of 90/40 mmHg.
Describe your management. Include in your answer how she is likely to deteriorate, and what general and specific therapies you would employ as her condition worsens.
Overarching Statement
This is a significant overdose of a non-dihydropyridine CCB, which would result in both vasodilatation and decreased inotropy/chronotropy. She already has symptomatic hypotension and bradycardia, which is likely to deteriorate and be prolonged due to the sustained release preparation ingested.
Immediate resuscitation –
Gastrointestinal decontamination-
Early contact with Poisons Information Centre (or equivalent) for advice.
Lipid “sink” therapy
Specific therapies - Simultaneous rather than stepwise therapy in this case given severity of CCB poisoning.
Marks were allocated more for specific management strategies than general resuscitation. Drug doses were not required.
Mention of Lipid Sink therapy essential to score greater than 4 marks
Expected pattern of deterioration
Specific management
General supportive management
Barrow, P. M., P. L. Houston, and D. T. Wong. "Overdose of sustained-release verapamil." BJA: British Journal of Anaesthesia 72.3 (1994): 361-365.
Mégarbane, Bruno, et al. "Predictors of mortality in verapamil overdose: usefulness of serum verapamil concentrations." Basic & clinical pharmacology & toxicology 108.6 (2011): 385-389.
Henry, Philip D. "Comparative pharmacology of calcium antagonists: nifedipine, verapamil and diltiazem." The American journal of cardiology 46.6 (1980): 1047-1058.
Doyon, Suzanne, and James R. Roberts. "The use of glucagon in a case of calcium channel blocker overdose." Annals of emergency medicine 22.7 (1993): 1229-1233.
Isbister, G. K. "Delayed asystolic cardiac arrest after diltiazem overdose; resuscitation with high dose intravenous calcium." Emergency medicine journal 19.4 (2002): 355-357.
Proano, Larry, William K. Chiang, and Richard Y. Wang. "Calcium channel blocker overdose." The American journal of emergency medicine 13.4 (1995): 444-450.
Engebretsen, Kristin M., et al. "High-dose insulin therapy in beta-blocker and calcium channel-blocker poisoning." Clinical toxicology 49.4 (2011).
Varpula, Tero, et al. "Treatment of serious calcium channel blocker overdose with levosimendan, a calcium sensitizer." Anesthesia & Analgesia 108.3 (2009): 790-792.
Frierson, John, et al. "Refractory cardiogenic shock and complete heart block after unsuspected verapamil‐sr and atenolol overdose." Clinical cardiology 14.11 (1991): 933-935.
Garg, Suneel K., et al. "Management of life-threatening calcium channel blocker overdose with continuous veno-venous hemodiafiltration with charcoal hemoperfusion." Indian journal of critical care medicine: peer-reviewed, official publication of Indian Society of Critical Care Medicine 18.6 (2014): 399.
Doepker, Bruce, et al. "High-dose insulin and intravenous lipid emulsion therapy for cardiogenic shock induced by intentional calcium-channel blocker and beta-blocker overdose: a case series." The Journal of emergency medicine 46.4 (2014): 486-490.
Cave, Grant, and Martyn Harvey. "Intravenous lipid emulsion as antidote beyond local anesthetic toxicity: a systematic review." Academic Emergency Medicine 16.9 (2009): 815-824.
Outline how the pathophysiological changes in septic shock affect the pharmacokinetlcs and pharmacodynamics of commonly used antimicrobials.
The major changes in pharmacokinetic parameters of critically ill patients include alterations in volume of distribution (Vd) and clearance (Cl). Subsequently, these alterations affect the concentrations of antimicrobials in the body and the extent to which they are cleared.
The Vd is the volume in which the total amount of drug would have to be evenly distributed in to equal the same concentration as in the plasma. The toxins produced by various bacteria often lead to endothelial damage and result in increased capillary permeability. This leads to the phenomenon of “third spacing” where fluid shifts into the interstitial space from the intravascular space. These fluid shifts will increase the Vd of hydrophilic antimicrobials. Generally speaking, hydrophilic antimicrobials have a low Vd and therefore are greatly affected by these fluid shifts. Since lipophilic antimicrobials have a larger Vd, they typically distribute further into tissues and are less affected by these fluid shifts. Patients in the ICU often have hypotension as a result of septic shock, which requires the administration of fluid boluses. Additionally, heart failure and renal failure lead to more oedematous states where patients can retain large amounts of fluid. These situations also lead to increases in Vd of hydrophilic drugs.
Changes in protein binding can also have a substantial effect on the Vd, especially for drugs that are highly protein bound. Only unbound or free drug is microbiologically active. Hypoalbuminemia in critically ill patients can result in decreased binding of drugs and subsequently higher free concentrations of drugs. While free drug will distribute into tissues, critically ill patients often have greater amounts of fluid in the interstitial space causing the antimicrobial concentrations in the tissues to remain low.
The administration of large volumes of fluid and use of vasopressors leads to a hypermetabolic state in which cardiac output and glomerular filtration rate are increased. The term often used to describe this enhanced elimination is augmented renal clearance. These physiological changes affect the clearance of drugs and can lead to sub-therapeutic levels of antimicrobials that are typically cleared by the kidneys. In contrast, decreased organ perfusion in the presence of end organ damage can lead to kidney and/or liver failure in which concentrations of these antimicrobials would be increased. Inadequate clearance or metabolism of these drugs would lead to accumulation and potential toxicity. Typically, equations such as Cockroft-Gault are used to estimate renal function; however, these are often not good predictors of renal function in critically ill patients due to the acute and rapid changes such patients often experience. Since many antimicrobials are dosed based on renal function it is even more challenging to ensure adequate doses are being administered. The most accurate way to calculate renal function is the use of 8- or 12-hour creatinine collections. In situations where renal replacement therapy is utilized, careful consideration of timing and supplemental dosing post-dialysis would be needed depending on the antimicrobial agent
Marks were awarded to answers which dealt with the following:
This question is identical to Question 10 from the second paper of 2015. An SEO-wrecking cut and paste is therefore permissible here, considering especially that the college reproduced their comments verbatim. In essence, just like last time, the examiners ask about septic shock, but then go on to discuss pharmacological changes which are common to all critically ill patients.
These are as follows:
Pharmacokinetic changes:
Pharmacodynamic changes:
As such, this model answer to a question about sepsis would have also answered Question 1 from the first paper of 2000, which asks about pharmacological changes in critical illness in a broader sense. There are a few pharmacological peculiarities which develop exclusively (or almost exclusively) in the context of sepsis, and these are summarised below using the excellent article by De Paepe et al (2002)
Change to pharmacology which are unique to sepsis and septic shock
Craig, William A. "Pharmacokinetic/pharmacodynamic parameters: rationale for antibacterial dosing of mice and men." Clinical infectious diseases (1998): 1-10.
Ulldemolins, Marta, et al. "Antibiotic dosing in multiple organ dysfunction syndrome." CHEST Journal 139.5 (2011): 1210-1220.
Trotman, Robin L., et al. "Antibiotic dosing in critically ill adult patients receiving continuous renal replacement therapy." Clinical infectious diseases 41.8 (2005): 1159-1166.
Drusano, George L. "Antimicrobial pharmacodynamics: critical interactions of'bug and drug'." Nature Reviews Microbiology 2.4 (2004): 289-300.
De Paepe, Peter, Frans M. Belpaire, and Walter A. Buylaert. "Pharmacokinetic and pharmacodynamic considerations when treating patients with sepsis and septic shock." Clinical pharmacokinetics 41.14 (2002): 1135-1151.
Piafsky, Kenneth M., et al. "Increased plasma protein binding of propranolol and chlorpromazine mediated by disease-induced elevations of plasma α1 acid glycoprotein." New England Journal of Medicine 299.26 (1978): 1435-1439.
Muller, Claudia M., et al. "Nitric oxide mediates hepatic cytochrome P450 dysfunction induced by endotoxin." The Journal of the American Society of Anesthesiologists 84.6 (1996): 1435-1442.
List the one pharmacological intervention for each of the following medications in the context of toxic ingestion leading to haemodynamic collapse. Outline the rationale for use of the pharmacological intervention including the mechanism of action.
a) Digoxin (25% marks)
b) Tricyclic anti-depressants (25% marks)
c) Beta blockers (25% marks)
d) Lignocaine (25% marks)
Not available.
This is not a new theme, but a new way of asking about the same (important) theme, a laudable development from the viewpoint of good assessment design. Beta blocker overdose digoxin toxicity and tricyclic antidepressant overdose are covered in detail elsewhere, and local anaesthetic toxicity is one of the syllabus items from the First Part exam, but it is good to see it migrate into the Fellowship papers because - let's face it - a first year ICU trainee is not going to be left to manage local anaesthetic toxicity with "haemodynamic collapse" on their own, i.e. one might argue that these topics belong in a senior curriculum.
Digoxin: Digoxin-specific Fab fragments are used as a "pharmacological intervention", and the article in UpToDate recommends that digoxin antibodies be used in every poisoning, even those who do not present with "haemodynamic collapse". Incidentally, that's obviously a colloquialism without any sort of a precise AHA/ESC definition, but we can let that slide because most intensivists will intuitively grasp it and relate. For example, in the case of digoxin, "haemodynamic collapse" looks like life-threatening arrhythmias and bradycardia.
Rationale for digoxin-specific Fab fragments in digoxin overdose:
Rationale for sodium bicarbonate in tricyclic antidepressant overdose:
Rationale for high dose insulin euglycaemic therapy in beta-blocker overdose:
Rationale for lipid infusion in local anaesthetic toxicity:
UpToDate has a nice article about digoxin toxicity..
Williamson, Kristin M., et al. "Digoxin toxicity: an evaluation in current clinical practice." Archives of internal medicine 158.22 (1998): 2444-2449.
Chan, B. S. H., and N. A. Buckley. "Digoxin-specific antibody fragments in the treatment of digoxin toxicity." Clinical Toxicology 52.8 (2014): 824-836.
Bou-Abboud, Elias, and Stanley Nattel. "Molecular mechanisms of the reversal of imipramine-induced sodium channel blockade by alkalinization in human cardiac myocytes." Cardiovascular research 38.2 (1998): 395-404.
Hoffman, Jerome R., et al. "Effect of hypertonic sodium bicarbonate in the treatment of moderate-to-severe cyclic antidepressant overdose." The American journal of emergency medicine 11.4 (1993): 336-341.
Engebretsen, Kristin M., et al. "High-dose insulin therapy in beta-blocker and calcium channel-blocker poisoning." Clinical toxicology (2011).
Christie, Linsey E., John Picard, and Guy L. Weinberg. "Local anaesthetic systemic toxicity." Bja Education 15.3 (2015): 136-142.
Ok, Seong-Ho, et al. "Lipid emulsion for treating local anesthetic systemic toxicity." International journal of medical sciences 15.7 (2018): 713.